Wikipedia:Reference desk/Science and User talk:SchfiftyThree: Difference between pages

From Wikipedia, the free encyclopedia
(Difference between pages)
Content deleted Content added
Smaug (talk | contribs)
 
No edit summary
 
Line 1: Line 1:
<small>'''''w w w . A n o n T a l k . c o m'''''</small>
[[Category:Non-talk pages that are automatically signed]][[Category:Science]]{{Wikipedia:Reference desk/header|WP:RD/S}}


{{User:SchfiftyThree/Navigation bar}}
{{Wikipedia:Reference_desk/Archives/Science/2008 October 4}}


{|align="center" width="160px" style="background-color: transparent;"
{{Wikipedia:Reference_desk/Archives/Science/2008 October 5}}
|-
|<div>
{{boxboxtop| }}
<div class="NavFrame" style="padding: 0; border-style: none;">
<div class="NavFrame" style="border-style: none; padding: 0;">
<div class="NavHead" style="background: #ffffff; text-align:left; text-style: normal;">
Vandalism Counter</div><div class="NavContent" style="display: none;">
{{User:UBX/user talk vandalized|24}}
</div>
{{boxboxbottom}}
</div>
|}


{{archive box|[[/Archive 1|Archive 1]]</br> [[/Archive 2|Archive 2]]</br> [[/Archive 3|Archive 3]]</br> [[/Archive 4|Archive 4]]</br> [[/Archive 5|Archive 5]]</br> [[/Archive 6|Archive 6]]}}
{{Wikipedia:Reference_desk/Archives/Science/2008 October 6}}


==Speedy deletion of [[:H-( miss a few )-R]]==
= October 7 =
[[Image:Ambox warning_pn.svg|48px|left]] A tag has been placed on [[:H-( miss a few )-R]], requesting that it be speedily deleted from Wikipedia. This has been done under [[WP:CSD#G1|section G1 of the criteria for speedy deletion]], because the page appears to have no meaningful content or history, and the text is unsalvageably incoherent. If the page you created was a test, please use the [[Wikipedia:Sandbox|sandbox]] for any other experiments you would like to do. Feel free to leave a message on my talk page if you have any questions about this.


If you think that this notice was placed here in error, you may contest the deletion by adding <code>{{tl|hangon}}</code> to '''the top of [[:H-( miss a few )-R|the page that has been nominated for deletion]]''' (just below the existing speedy deletion or "db" tag), coupled with adding a note on '''[[ Talk:H-( miss a few )-R|the talk page]]''' explaining your position, but be aware that once tagged for ''speedy'' deletion, if the article meets the criterion it may be deleted without delay. Please do not remove the speedy deletion tag yourself, but don't hesitate to add information to the article that would would render it more in conformance with Wikipedia's policies and guidelines. Lastly, please note that if the article does get deleted, you can contact [[:Category:Wikipedia administrators who will provide copies of deleted articles|one of these admins]] to request that a copy be emailed to you. <!-- Template:Db-nonsense-notice --> <!-- Template:Db-csd-notice-custom --> [[User:Largoplazo|Largo Plazo]] ([[User talk:Largoplazo|talk]]) 00:26, 16 September 2008 (UTC)
== Quantum: Difference between an operator and a measurement ==
:That was because of a somewhat automatic error. The whitelist page got moved to a vandal title, thus creating the title the Grawp-vandal made. [[User:SchfiftyThree|<font color="4169E1">'''Schfifty'''</font>]][[User talk:SchfiftyThree|<font color="DC143C">'''Three'''</font>]] 00:28, 16 September 2008 (UTC)


::Ouch! Cool, no problem. &#8212;[[User:Largoplazo|Largo Plazo]] ([[User talk:Largoplazo|talk]]) 00:39, 16 September 2008 (UTC)
Suppose there is a qubit whose state is
:<math>|\psi\rangle=\frac 1{\sqrt 3}|0\rangle+\frac{\sqrt 2}{\sqrt 3}|1\rangle=\begin{bmatrix}\tfrac 1{\sqrt 3}\\ \tfrac{\sqrt 2}{\sqrt 3}\end{bmatrix}</math>
After we measure the qubit, the state of the qubit will change from <math>|\psi\rangle</math> to <math>|1\rangle</math> with probability
:<math>\big|\langle 1|\psi\rangle\big|^2=\left(\frac{\sqrt 2}{\sqrt 3}\right)^2=\frac 2 3\approx 0.666\dots</math>.
This process is called [[Wave function collapse|wave function collapse]]. If <math>|1\rangle</math> is observed after the measurement, the qubit becomes
:<math>|\psi\rangle=|1\rangle</math>


== Your [[WP:NPW|NPWatcher]] application ==
Instead of measuring the qubit, a [[Hadamard gate]]
:<math>H=\frac 1{\sqrt 2}\begin{bmatrix}1&1\\1&-1\end{bmatrix}</math>
operates on the qubit will be
:<math>H|\psi\rangle=\frac 1{\sqrt 2}\begin{bmatrix}1&1\\1&-1\end{bmatrix}\begin{bmatrix}\tfrac 1{\sqrt 3}\\ \tfrac{\sqrt 2}{\sqrt 3}\end{bmatrix}=\frac 1{\sqrt 2}\begin{bmatrix}\tfrac{1+\sqrt 2}{\sqrt 3}\\ \tfrac{1-\sqrt 2}{\sqrt 3}\end{bmatrix}=\frac{1+\sqrt 2}{\sqrt 6}|0\rangle+\frac{1-\sqrt 2}{\sqrt 6}|1\rangle</math>
as I know, the process of the operation is 'not' a wave function collapse.


Dear SchfiftyThree,
My problem is why an operator acts on a qubit doesn't cause a wave function collapse? As I know, any subtle interaction with the qubit will cause the wave function to collapse. The Hadamard gate operator, which should be an apparatus, when acts on the qubit should also interact with the qubit. So how an operator can circumvent the wave function collapse? - [[User:Justin545|Justin545]] ([[User talk:Justin545|talk]]) 05:28, 7 October 2008 (UTC)


Thank you for applying for [[WP:NPW|NPWatcher]]!
:I'm a little rusty in my quantum theory, but if I remember correctly, it's not all interactions that cause wave-form collapse, only those actions that generate information. It is entirely possible to transform a wave-function without actually observing (i.e. gaining information from) said wave function. Take for example the most basic quantum experiment, the [[double-slit experiment]]. The slit certainly ''interacts'' with the beam of particles; as the beam hits the slits, an interference pattern immerges, this is a result of the slits "transforming" the beam of particles via interaction. The wave function collapse occurs only when you try to gain information about the particle's location, for example, by placing a charge detector at one of the slits. As long as no information is obtained on the system, it goes on behaving as an uncollapsed wave function. Once the detector is placed, information is extracted, and the wave function collapses, resulting in an uninterferred double beam... The Hadamard gate must operate in the same way; it performs a transformation on the qubit blindly (that is, without observing the state of the qubit). Such an action is not philosophically that hard to understand. Imagine a blind man turning coins over. You hand the man a coin, and he simply reverses the face of the coin. He can perform the operation (turning heads to tails and tails to heads) perfectly every time, even if he doesn't know which states the coins are in before he flips them. To take this on a quantum scale, he's able to perform a transformation on the wave function, without causing any waveform collapse. He's made no observations of the bits of information he's transforming, he's just doing the transformation. A Hadamard gate must work on this level. --[[User:Jayron32|Jayron32]].[[User talk:Jayron32|<small>talk</small>]].[[Special:Contributions/Jayron32|<small>contribs</small>]] 05:47, 7 October 2008 (UTC)
You've been approved to use it. Before you run the program, please check the changelog on the application page to see if there is a newer release (or just add the main page ([[User:Martinp23/NPWatcher{{!}}here]]) to your watchlist). Report any bugs or feature suggestion [[User_talk:Martinp23/NPWatcher{{!}}here]]. If you need help, feel free to contact me or join [irc://irc.freenode.net/NPWatcher NPWatcher].


—[[user:aitias|αἰτίας]] ''•''[[User talk:Aitias|''discussion'']]''•'' 15:37, 18 September 2008 (UTC)
::It could be difficult to define the term 'gaining information from'. Cite form the article [[Quantum computer]]:


==Thank you==
:::''One major problem is keeping the components of the computer in a coherent state, as the slightest interaction with the external world would cause the system to '''decohere'''. This effect causes the unitary character (and more specifically, the invertibility) of quantum computational steps to be violated. Decoherence times for candidate systems, in particular the transverse relaxation time T2 (terminology used in NMR and MRI technology, also called the dephasing time), typically range between nanoseconds and seconds at '''low temperature'''.''
Hi SchfiftyThree. I would like to thank you for your support in my RfA and the confidence expressed thereby. It is very much appreciated. :) The RfA was closed as successful with 73 supports, 3 opposes and 4 neutral. I would especially like to thank [[User:WBOSITG|WBOSITG]] for nominating me. Best wishes and thanks again, —[[user:aitias|αἰτίας]] ''•''[[User talk:Aitias|''discussion'']]''•'' 22:51, 18 September 2008 (UTC)


==My RfA==
::I think the 'decohere' is actually the wave function collapse. It seems that even the 'temperature' is also a way to interact with the qubit and it is not clear to me how an environmental temperature makes information gaining... maybe what you mean is that a measurement is an irreversible operation whereas a gate operation is a reversible operation. Indeed, the action of a Hadamard gate is a reversible operation since there is no information lost during the operation of the Hadamard gate. And saying that a measurement is an irreversible operation is just my suspicion since some information is lost and gained by the observer during a measurement. By thinking the gate operation as a transform as you said make it more understandable to me. And now I know there are some interactions don't collapse the wave function. - [[User:Justin545|Justin545]] ([[User talk:Justin545|talk]]) 07:23, 7 October 2008 (UTC)
{|style="background-color: #E6E6FA; padding: 3px; border: 1px solid #888;"
|[[Image:Hurricane Kate (2003)- Good pic.jpg|100px]]
|style="background-color: #def; padding: 0 10px 0 10px; border: 1px solid #888;"|Thank you for supporting me in my RfA, which passed with a count of (154/3/2). I appreciate the community's trust in me, and I will do my best to be sure it won't regret handing me the mop. I am honored by your trust and your support. Again, thank you. &ndash;[[User:Juliancolton|Juliancolton]] [[User talk:Juliancolton|<font color="#66666"><sup>'''T'''ropical</sup></font>]] [[Special:contributions/Juliancolton|<font color="#66666"><sup>'''C'''yclone</sup></font>]] 18:28, 27 September 2008 (UTC)
|}


==AfD nomination of Blood Meridian (film)==
:::Unless the qubit is lost due to interaction with other particles; at high temperatures there are bound to be more particles moving faster and thus more interference on the qubit; there may be some threshold temperature where the system becomes so inefficient due to losses that it stops "working." --[[User:Jayron32|Jayron32]].[[User talk:Jayron32|<small>talk</small>]].[[Special:Contributions/Jayron32|<small>contribs</small>]] 11:29, 7 October 2008 (UTC)
[[Image:Ambox warning pn.svg|48px|left]]An article that you have been involved in editing, [[Blood Meridian (film)]], has been listed for [[Wikipedia:Deletion policy|deletion]]. If you are interested in the deletion discussion, please participate by adding your comments at [[Wikipedia:Articles for deletion/Blood Meridian (film)]]. Thank you. <small>Do you want to [[Template:Bots#Message notification opt out|opt out]] of receiving this notice?</small><!-- Template:Adw --> [[User:Schuym1|<span style="color:#0000f1">Schuy</span><span style="color:#00FFFF">m</span><span style="color:#FFBF00">1</span> <small>]]([[User talk:Schuym1|<font style="color:green">talk</font>]])</small> 17:16, 28 September 2008 (UTC)
*Actaully, it said it's rumored. AFD will continue. [[User:Schuym1|<span style="color:#0000f1">Schuy</span><span style="color:#00FFFF">m</span><span style="color:#FFBF00">1</span> <small>]]([[User talk:Schuym1|<font style="color:green">talk</font>]])</small> 01:16, 29 September 2008 (UTC)


== [[User:II MusLiM HyBRiD II]]'s navbar ==
:::Definitely some interactions don't collapse the wave function, otherwise a proton, for example, couldn't be a quantum particle since it consists of more fundamental particles constantly interacting with each other.
:::See [[quantum decoherence]] for more on that subject. It is effectively the same thing as wavefunction collapse and people are likely to use the terms interchangeably.
:::Measurement/decoherence is all about copying. An example of copying is a transition from <math>\alpha|0\rangle + \beta|1\rangle</math> to <math>\alpha|0\rangle|0\rangle + \beta|1\rangle|1\rangle</math>. This is different from [[no-cloning theorem|cloning]], which would take <math>\psi</math> to <math>\psi^2</math>, e.g. <math>(\alpha|0\rangle + \beta|1\rangle)^2 = \alpha^2|0\rangle|0\rangle + \cdots</math>. Cloning isn't possible, but copying is possible. The key difference between them is that copying is basis-dependent. The example I gave above was copying with respect to the computational basis <math>\{|0\rangle,|1\rangle\}</math>. Copying with respect to the dual basis <math>|{\pm}\rangle = \tfrac1{\sqrt2}(|0\rangle \pm |1\rangle)</math> takes
::::<math>\alpha|{+}\rangle + \beta|{-}\rangle</math> to <math>\alpha|{+}\rangle|{+}\rangle + \beta|{-}\rangle|{-}\rangle</math>,
:::which is the same as taking
::::<math>\alpha|{0}\rangle + \beta|{1}\rangle</math> to <math>\tfrac\alpha{\sqrt2}(|00\rangle+|11\rangle) + \tfrac\beta{\sqrt2}(|01\rangle+|10\rangle)</math>,
:::if I calculated right—at any rate it's a different operation. If you copy a qubit (with respect to a basis) and put the copy somewhere where it's unavailable to you, the effect on the original qubit is exactly as though you'd measured it (with respect to that basis). But if the copy ever becomes accessible to you again, you can use it to "undo the measurement," so it's not a real measurement. Measurements only become definite when they can no longer be undone, e.g. because the copy has been amplified into a macroscopic thermodynamically irreversible state change like a flash from an LED.
:::Reply composed in a hurry and I may not be able to reply again for a few days. -- [[User:BenRG|BenRG]] ([[User talk:BenRG|talk]]) 11:44, 7 October 2008 (UTC)


I suppose you realise that everything here is licensed under the GFDL, and as such he is perfectly within rights to use it as he has? Not that I agree with taking stuff from others' userpages without asking, I'm just talking from a legal perspective. <small><span style="border:1px solid black;padding:1px;">[[User:Asenine|<i><font style="color:white;background:#4682b4;font-family:sans-serif;">'''&nbsp;Ase'''</font></i>]][[User talk:Asenine|<i><font style="color:white;background:#4682b4;font-family:sans-serif;">'''nine&nbsp;'''</font></i>]]</span></small> 21:47, 29 September 2008 (UTC)
A short mathematical answer: [[Self-adjoint operator]]s correspond to measurements. [[Unitary operator]]s correspond to state changes without any wave function collapse. [[User:Icek|Icek]] ([[User talk:Icek|talk]]) 21:32, 8 October 2008 (UTC)
:Also:-


**Girlfriends age ________?
== Noisy laptop fan ==
**My IQ ________?
**You must pay ________? <small><span style="border:1px solid black;padding:1px;">[[User:Asenine|<i><font style="color:white;background:#4682b4;font-family:sans-serif;">'''&nbsp;Ase'''</font></i>]][[User talk:Asenine|<i><font style="color:white;background:#4682b4;font-family:sans-serif;">'''nine&nbsp;'''</font></i>]]</span></small> 21:50, 29 September 2008 (UTC)
::I get that, except I'm two numbers behind... :-) [[User:SchfiftyThree|<font color="4169E1">'''Schfifty'''</font>]][[User talk:SchfiftyThree|<font color="DC143C">'''Three'''</font>]] 21:59, 29 September 2008 (UTC)


:::Yeah, how could i say this. Im sorry, and i didnt copy it from you though. It was from another user, who left wiki, so i thought i could use his. I didnt want to cause any problems, and i dotn see how taking a nav bar from another user is that important?Anyways, sorry. [[User:II MusLiM HyBRiD II |<span style='font-family:"Arial Black";color:white;background:blue'>II MusLiM </span><span style='font-family:"Arial Black";color:yellow;background:black'>HyBRiD II </span>]] 22:33, 29 September 2008 (UTC)
Why do they (fans) do it (noise)? Is a silent laptop fan possible?[[User:Mr.K.|Mr.K.]] [[User_talk:Mr.K.|(talk)]] 11:06, 7 October 2008 (UTC)
::::You don't have to apologise for having a similar navigation bar like mine. I just left the message because I ''noticed'' it. You may continue to keep it the way you like it, though. One suspicion note: I think it is very similar to mine, but having made a few changes (e.g. background colour). The image to your userbox page has a URL as "User:II MusLiM HyBRiD II/Userbox Info", whereas the Userbox Info is the main title of my userbox subpage. Who knows? It might have been from mine, but may still keep it the way you like it. Regards, [[User:SchfiftyThree|<font color="4169E1">'''Schfifty'''</font>]][[User talk:SchfiftyThree|<font color="DC143C">'''Three'''</font>]] 22:38, 29 September 2008 (UTC)
:A (laptop) fan moves air, using an electric engine. Air flow creates sound, and the engine can't be perfectly silent either, so: no, a silent laptop fan isn't possible. There are however some new, interesting ways of cooling coming about, some of which may be silent. Someone please fill me in here :) -- [[User:Aeluwas|Aeluwas]] ([[User talk:Aeluwas|talk]]) 11:08, 7 October 2008 (UTC)
:::::Okays, Thanks~ :) [[User:II MusLiM HyBRiD II |<span style='font-family:"Arial Black";color:white;background:blue'>II MusLiM </span><span style='font-family:"Arial Black";color:yellow;background:black'>HyBRiD II </span>]] 22:57, 29 September 2008 (UTC)
:::There is liquid cooling, but that would be for rack-mount mega-chips (stacked processors have a higher power density than a nuclear reactor), not laptops; and the liquid would still need cooling. See [http://www.economist.com/science/tq/displaystory.cfm?story_id=11999317 this ''Economist'' article] (and I can email it if you can't see it). [[User:Franamax|Franamax]] ([[User talk:Franamax|talk]]) 22:13, 8 October 2008 (UTC)
::Actually, there are ways of generating air flow without any moving parts, for example see [[Air ionizer]]; it operates similar to a [[mass spectrometer]]; air molecules can be ionized via say, a negatively charged plate, and accelerated via a pair of positively charged plates. Many air molecules will simply gain electrons at one plate and lose them back at the other (or visa-versa; I am not positive on the specific mechanics of the situation), however, some will "overshoot" the second plate, and also drag many "non-charged" molecules with them, resulting in net air flow. However, the method is quite innefficient, especially on a scale small enough to fit into a laptop, and I suspect that in terms of net air flow for, for both its size and wattage, a simple mechanical fan is far more efficient. --[[User:Jayron32|Jayron32]].[[User talk:Jayron32|<small>talk</small>]].[[Special:Contributions/Jayron32|<small>contribs</small>]] 11:24, 7 October 2008 (UTC)


== Revert ==
The engine could be so silent like the HDD. Moving air doesn't have to be noisy. I suppose it is noisy only if it generates turbulence right? So, a silent laptop fan should be possible... [[User:Mr.K.|Mr.K.]] [[User_talk:Mr.K.|(talk)]] 11:38, 7 October 2008 (UTC)


That wasn't a user page, it was an article. You can't create an article of yourself unless you're really famous. If that were the case, we would have a good 4.5 billion bios in Wikipedia. Regards,<font face="cursive">''[[User:Vinsfan368|<span style="color:#00FFFF">Vinson</span>]]''</font> 23:35, 29 September 2008 (UTC)
:But laminar airflow may not be terribly efficient at moving heat around, as it may not move all the air and probably wouldn't move fast enough. [[Special:Contributions/130.88.64.189|130.88.64.189]] ([[User talk:130.88.64.189|talk]]) 12:29, 7 October 2008 (UTC)
:Hmm...you might have a point there. I think the user might not have created the article about himself, but saw the A7 notice and blanked the page. I did not even know how the edit would be considered "unconstructive." [[User:SchfiftyThree|<font color="4169E1">'''Schfifty'''</font>]][[User talk:SchfiftyThree|<font color="DC143C">'''Three'''</font>]] 23:40, 29 September 2008 (UTC)


== Re: first-edit day ==
:The other common means of making a fan quieter is to spin it at a lower RPM. Of course, the direct consequence is that you move less air, reducing the cooling effectiveness. In a desktop system, that just means that you make the fan larger to compensate. With space at a premium in laptops, this solution is less effective. &mdash; [[User talk:Lomn|Lomn]] 12:59, 7 October 2008 (UTC)


::A related option is to reduce your CPU usage, which reduces the laptop's need for cooling, which should let the fan run slower and less frequently. [[User:Plasticup|<b><font color="#0080FF">Plasticup</font></b>]] [[User_Talk:Plasticup |<font color="#2A8E82"><sup><small>T</small></sup></font>]]/[[Special:Contributions/Plasticup|<font color="#2A8E82"><small>C</small></font>]] 15:15, 7 October 2008 (UTC)
Thank you! I'd completely forgotten about it. Has it really been two ''years?'' Wow. [[User:J.delanoy|<font color="green">J'''.'''delanoy</font>]][[User Talk:J.delanoy|<sup><font color="red">gabs</font></sup>]][[Special:Contributions/J.delanoy|<font color="blue"><sub>adds</sub></font>]] 04:52, 2 October 2008 (UTC)


== Foxy Loxy's RfA ==
:::Often, reducing the CPU usage is not an option, since I use the laptop with a purpose. [[User:Mr.K.|Mr.K.]] [[User_talk:Mr.K.|(talk)]] 15:24, 7 October 2008 (UTC)


::::Amen to that! [[User:Plasticup|<b><font color="#0080FF">Plasticup</font></b>]] [[User_Talk:Plasticup |<font color="#2A8E82"><sup><small>T</small></sup></font>]]/[[Special:Contributions/Plasticup|<font color="#2A8E82"><small>C</small></font>]] 00:28, 8 October 2008 (UTC)
Hello, this message is to inform you that [[User:Foxy Loxy]] has restarted their RfA. The new discussion is located at [[Wikipedia:Requests for adminship/Foxy Loxy 2]]. [[User:GlassCobra|<font color="002bb8">Glass</font>]]'''[[User talk:GlassCobra|<font color="002bb8">Cobra</font>]]''' 10:01, 3 October 2008 (UTC)
::To remove a particular amount of heat per minute, you must move a particular amount of air per minute, often measured in CFM. A fan with a smaller diameter must spin faster for the same CFM. The higher speed results in a higher amount of vorticity at the blade ends, and this is where most of the noise is generated. A laptop cannot accomodate a large-diameter fan, so the fan must spin very fast and is therefore very noisy. The same is true for 1U rack-mount servers. As an example of the difference diameter makes, consider the difference between a 20" diameter window fan and a 3" diameter "muffin" fan in a desktop computer. The window fan at its lowest speed moves much more air than the muffin fan at its normal niosy speed, but is virtually silent.The motor contributes very little noise in either case. -[[User:Arch dude|Arch dude]] ([[User talk:Arch dude|talk]]) 00:41, 8 October 2008 (UTC)


== AllieKontor ==
== Question (Goliath Beetle) ==


I believe you beat me to a previous attack page this morning/evening, so we're even :P. [[User:Ironholds|<b style="color:#D3D3D3">Ir</b><b style="color:#A9A9A9">on</b><b style="color:#808080">ho</b>]][[User talk:Ironholds|<b style="color:#696969">ld</b><b style="color:#000">s</b>]] 04:04, 5 October 2008 (UTC)
What is the main predator to the Goliath Beetle? <span style="font-size: smallest;" class="autosigned">—Preceding [[Wikipedia:Signatures|unsigned]] comment added by [[Special:Contributions/204.210.238.42|204.210.238.42]] ([[User talk:204.210.238.42|talk]]) 11:54, 7 October 2008 (UTC)</span><!-- Template:UnsignedIP --> <!--Autosigned by SineBot-->
:As a matter of fact, I don't even recall tagging a page for CSD today, plus the article has already been deleted. [[User:SchfiftyThree|<font color="4169E1">'''Schfifty'''</font>]][[User talk:SchfiftyThree|<font color="DC143C">'''Three'''</font>]] 04:06, 5 October 2008 (UTC)
::Must have been someone else then :S. In that case i'll accept the appreciation of my greatness, or at least that of my uni internet connection :P [[User:Ironholds|<b style="color:#D3D3D3">Ir</b><b style="color:#A9A9A9">on</b><b style="color:#808080">ho</b>]][[User talk:Ironholds|<b style="color:#696969">ld</b><b style="color:#000">s</b>]] 04:13, 5 October 2008 (UTC)


== Thanks for signing my guestbook ==
: I'm not sure it has a main predator. There are more than enough predators and omnivores in tropical Africa that would not think twice before dining on a huge delicious beetle larva. [[Mandrill]] comes to mind as a very likely predator, but I couldn't find any specific data. Besides, Mandrill habitat extent is much smaller than that of ''[[Goliathus]] sp.'' . Sorry. --[[User:Dr Dima|Dr Dima]] ([[User talk:Dr Dima|talk]]) 17:13, 7 October 2008 (UTC)


{{User:LAAFan/Barnstar userbox}}
== How does fruit get "bruised"? ==

Just curious how fruits like apples and bananas get "bruised"? Aren't they dead? How does impact against the surface affect the fruit below the skin? --[[Special:Contributions/70.167.58.6|70.167.58.6]] ([[User talk:70.167.58.6|talk]]) 16:00, 7 October 2008 (UTC)

:I'm not sure it's accurate to say they're dead. They are still chemically active, otherwise they wouldn't be able to ripen after being picked. I think the bruising is caused by breaking the cell walls so the individual cells die, but I can't guarantee it. I'm sure a botanist will be along shortly. --[[User:Tango|Tango]] ([[User talk:Tango|talk]]) 16:07, 7 October 2008 (UTC)

::Not a botanist, but Tango has the right idea. Inside of plant cells are chemicals which oxidize upon exposure to air. If the cell remains intact, nothing happens. However, cutting the fruit or striking it can cause the cell walls of the cells to break, exposing the chemicals to air, and causeing a change in both color and texture. Incidentally, bruised fruit is perfectly healthy and there's nothing at all wrong with eating it. You abuse the fruit much more when you chew and swallow the fruit anyways; the color change is not a sign of bacterial growth in any way, its merely a sign of physical damage. --[[User:Jayron32|Jayron32]].[[User talk:Jayron32|<small>talk</small>]].[[Special:Contributions/Jayron32|<small>contribs</small>]] 16:47, 7 October 2008 (UTC)

: You may want to look at [http://postharvest.ucdavis.edu/datastorefiles/234-51.pdf this]. --[[User:Dr Dima|Dr Dima]] ([[User talk:Dr Dima|talk]]) 19:18, 7 October 2008 (UTC)

:Fresh animal products consists of all dead cells when you buy them but fruits fruits and vegetables are all alive. The "bruise" consists of plant cells that are dead. [[User:Sjschen|Sjschen]] ([[User talk:Sjschen|talk]]) 03:51, 8 October 2008 (UTC)

::So is it possible for bruises to grow? Will a bruise "infect" undamaged cells making it more unattractive? Do bruised areas rot faster? --[[Special:Contributions/71.158.222.207|71.158.222.207]] ([[User talk:71.158.222.207|talk]]) 03:59, 8 October 2008 (UTC)
:::Yes, bruises will grow - the [[lysis|lysed]] cells burst in the original insult will release oxidizing enzymes which will continue to degrade the contents of the fruit, in part by lysing more cell membranes. And yes, as more cells contents are spilled out, they will be more susceptible to bacterial invasion. This is why the little bruise spot on your tomato when you bought it gets bigger and gets a grey mold at the centre. [[User:Franamax|Franamax]] ([[User talk:Franamax|talk]]) 05:54, 8 October 2008 (UTC)
:Unquestionably the best resource on this is ''On Food and Cooking: The Science and Lore of the Kitchen'' H. McGee ISBN 978-0-684-80001-1.[http://books.google.com/books?id=6S--wG3pYZoC&q=on+food+and+cooking&dq=on+food+and+cooking&pgis=1] It explains all this stuff ''and'' tells you how to get your eggs right! :) [[User:Franamax|Franamax]] ([[User talk:Franamax|talk]]) 06:18, 8 October 2008 (UTC)

== Grass Seed Lifespan ==

I have had a bag of lawn grass seed in my garage for over 4 years. Over that time it has been exposed to -20C to +35C degrees. Would it still be alive and able to grow after all? <span style="font-size: smaller;" class="autosigned">—Preceding [[Wikipedia:Signatures|unsigned]] comment added by [[Special:Contributions/151.123.128.250|151.123.128.250]] ([[User talk:151.123.128.250|talk]]) 16:40, 7 October 2008 (UTC)</span><!-- Template:UnsignedIP --> <!--Autosigned by SineBot-->

:Quite possibly. Many kinds of seeds are quite resilient. There are some seeds that have been shown to germinate after hundreds of years of dormancy. The best thing to do is to run a little experiment. Take a cup of dirt, put a small pinch of seeds in it, keep it moist and in direct sunlight (like the windowsill) and see what happens. The seeds, if still viable, should germinate within a few days. If they do, you probably have good seed. If it just sits there for a week and nothing happens, then you probably need to pitch them. --[[User:Jayron32|Jayron32]].[[User talk:Jayron32|<small>talk</small>]].[[Special:Contributions/Jayron32|<small>contribs</small>]] 16:44, 7 October 2008 (UTC)
::The normal rule is: If at least 75% of the seeds germinate, use as directed. If 50%-75% germinate, use double the normal amount. If less than 50% germinate, toss it and get a new bag. --[[User:Stephan Schulz|Stephan Schulz]] ([[User talk:Stephan Schulz|talk]]) 17:41, 7 October 2008 (UTC)

== calorie consumption per hour/ mile (cycling) ==

Hello Everyone,

i've been trying to work out how many calories I burn on my cycle to work but i'm struggling to get some relevant info (i've been on a few forums but they use 40 year olds as reference points -i'm not even sure if one burns more or less calories as one ages so this is less than helpful.) Anyway, i'd be grateful if any one has any ideas - its about 4.7 miles (according to google maps anyway) each way, along reasonably flat terrain -stopping and starting for traffic lights often. It takes me about 30 minutes on average. Also i'm a (reasonably fit) 24 year old man... Any help, or even some starting data, would be great...

Thanks, [[Special:Contributions/82.22.4.63|82.22.4.63]] ([[User talk:82.22.4.63|talk]]) 19:09, 7 October 2008 (UTC)

:While caloric intake is relatively easy to calculate, based on the nutritional content of the food in terms of ammounts of proteins, fats, and carbohydrates, its a different story for caloric output, in terms of exercise. Its going to vary WILDLY depending on how hard you are pedaling, your current body weight, your personal body chemistry and metabolism, the ambient temperature that day, your muscle mass, yada yada yada. The variables are almost too great to even think of all of them. Calculators that purport to determine how many calories you burn (for example, those found on treadmills or stationary bikes) are likely just WAGs. They may be based on, for example, the 50th percentile human, but variation from that ideal is likely so large that there is no way to assure they will be accurate for you. The easiest way to tell is to see if it has an effect on your own body: If you are losing body fat over time, you are likely burning more calories than you take in. If you are gaining body fat over time, you are likely consuming more than you burn. Take [[Michael Phelps]] for example. It is widely reported that he consumes 10,000 calories per day. That's roughly the amount of food 4 people would eat. And yet, he has almost no body fat, which means his level of activity has him burning at least that much. The funny thing is, another person, performing the same workout regimen as he does, may find himself gaining fat at that level of intake. Isolating WHICH activities are burning which calories is entirely a guessing game. The best thing that can be said is more activity always burns more calories, so more is always better. --[[User:Jayron32|Jayron32]].[[User talk:Jayron32|<small>talk</small>]].[[Special:Contributions/Jayron32|<small>contribs</small>]] 20:21, 7 October 2008 (UTC)

== Human feet vs. Chimp feet ==

Chimps' feet have thumb like digits on them so they can grab objects with their feet in a similar way to how we can grab things with our hands correct? So I was curious if our feet which do not have that ability, have any advantages over their feet. [[Special:Contributions/98.221.85.188|98.221.85.188]] ([[User talk:98.221.85.188|talk]]) 19:51, 7 October 2008 (UTC)

:Chimp feet are great for walking on branches, too. But they're lousy for running. (<s>See [[achilles tendon]]</s> not what I expected.) See [[Persistence hunting]]. [[User:Saintrain|Saintrain]] ([[User talk:Saintrain|talk]]) 20:07, 7 October 2008 (UTC)

::Chimps are also not bipedal like humans. Like some other animals (bears, for example) they can, relatively easily walk on two legs, but their preferred method of ground locomotion is "knuckle walking" on all fours, generally as a sort of side-shuffle. Chimps are not very efficient at walking flat footed, however, as noted above, they are VERY efficient at traveling in trees, having evolved in a forested habitat. Modern humans largely evolved in a savannah habitat, with few trees, and thus flat-footed walking gave them an advantage in that environment. Thumbs are an impediment to flat-footed walking, and so proto-humans with "modern human" style feet tended to predominate in that environment. --[[User:Jayron32|Jayron32]].[[User talk:Jayron32|<small>talk</small>]].[[Special:Contributions/Jayron32|<small>contribs</small>]] 20:26, 7 October 2008 (UTC)

::To expand on Jayron32, the human foot is specifically adapted to walking upright. The big advantage we have over chimps (for the bipedal environment) are the two [[arches of the foot]] which act --like springs-- as shock absorbers, distributing weight and allowing us to easily run upright. --[[User:Shaggorama|Shaggorama]] ([[User talk:Shaggorama|talk]]) 20:56, 7 October 2008 (UTC)

::I was talking a while back to someone who lost their big toe in a lawnmower accident and they said the loss of the toe made walking quite bit more difficult. So it looks like the toe might have been actively evolved to help with walking. [[User:Dmcq|Dmcq]] ([[User talk:Dmcq|talk]]) 00:30, 8 October 2008 (UTC)

== Fizzy drinks. ==

The aqueous carbon dioxide in fizzy drinks makes the drinks acidic (for some reason that I have forgotten), couldn't you just add an alkaline solution to the drink to neutralise it. Fizzy drinks are critisised for being bad for your teeth because they're acidic, this would get rid of that problem. Thanks.[[Special:Contributions/92.2.212.124|92.2.212.124]] ([[User talk:92.2.212.124|talk]]) 20:02, 7 October 2008 (UTC)

:Except that acidic compounds have a generally pleasant "sour" flavor. Alkali compounds have a bitter, soapy flavor. Eat a bar of soap, or drink some lemonade. Which do YOU prefer? --[[User:Jayron32|Jayron32]].[[User talk:Jayron32|<small>talk</small>]].[[Special:Contributions/Jayron32|<small>contribs</small>]] 20:31, 7 October 2008 (UTC)

::Also, its not the acid in sodas that is damaging to your teeth '''per se''' since it does not remain in contact with your teeth for very long. However, bacteria, which DO remain in contact with your teeth for a long time, will feed on the sugar in the soda, and produce an acidic waste product. Since these bacteria are essentially always there, the more sugar you give them, the more acid they make. Its this acid, which is held directly against the tooth for a long period of time, and not the acid that washes over the tooth which causes decay. Its the sugars in the soda that cause the decay, not the acid inherent in the [[carbonic acid]]. --[[User:Jayron32|Jayron32]].[[User talk:Jayron32|<small>talk</small>]].[[Special:Contributions/Jayron32|<small>contribs</small>]] 20:31, 7 October 2008 (UTC)

:::Thanks[[Special:Contributions/92.2.212.124|92.2.212.124]] ([[User talk:92.2.212.124|talk]]) 20:43, 7 October 2008 (UTC)

::: It is not [[carbonic acid]] that is responsible for the sour taste and the tooth weakening, it is [[phosphoric acid]] (Cola) or [[citric acid]]. [[User:Cacycle|Cacycle]] ([[User talk:Cacycle|talk]]) 03:28, 8 October 2008 (UTC)

From [http://www.thejcdp.com/issue036/owens/index_nlm.htm this article], it's due to a combination of acidity, buffering capacity (titratable acidity) and sugar content. [[User:Axl|<font color="#808000">'''Axl'''</font>]] <font color="#3CB371">¤</font> <small>[[User talk:Axl|<font color="#6B8E23">[Talk]</font>]]</small> 17:40, 8 October 2008 (UTC)

== The grayed tip of a banana ==

As a child I was told to bite off the usually grayed tip of a banana before eating it. What is it, though? Thanks. [[User:Imagine Reason|Imagine Reason]] ([[User talk:Imagine Reason|talk]]) 20:03, 7 October 2008 (UTC)
:Greyed tip? I assume this means pre-peeled bananas, but i wouldn't bite the peel of a banana. If this means a peeled banana, what grey tip? Anyway, I've never heard that.[[Special:Contributions/92.2.212.124|92.2.212.124]] ([[User talk:92.2.212.124|talk]]) 20:30, 7 October 2008 (UTC)
:I'm pretty sure it's banana. --[[User:Shaggorama|Shaggorama]] ([[User talk:Shaggorama|talk]]) 20:38, 7 October 2008 (UTC)
::Yes, it's the tip of a just-peeled banana. It's usually not white. C'mon, I thought that were the common experience. [[User:Imagine Reason|Imagine Reason]] ([[User talk:Imagine Reason|talk]]) 21:19, 7 October 2008 (UTC)

:::See [[vascular bundle]] and, specifically for bananas, [[phloem bundle]] (What!? No article!?). What you are referring to is the part of the banana where the bundles (strings) join and invert into the center of the fruit. -- [[User:Kainaw|<font color='#ff0000'>k</font><font color='#cc0033'>a</font><font color='#990066'>i</font><font color='#660099'>n</font><font color='#3300cc'>a</font><font color='#0000ff'>w</font>]][[User talk:Kainaw|&trade;]] 21:49, 7 October 2008 (UTC)

:You mean the dark bit at the far end from where you open it? I don't bite it off, it comes off very cleanly on its own if you push it sideways slightly. --[[User:Tango|Tango]] ([[User talk:Tango|talk]]) 23:17, 7 October 2008 (UTC)
::[[Wikipedia:WikiProject Countering systemic bias|Systemic bias]]! Seriously, you don't open a banana at the stem end? Weird. --[[User:TotoBaggins|Sean]] 22:43, 8 October 2008 (UTC)
:This sounds like bruising as described not far above. When the cells are damaged, they release enyzmes which oxidize the surrounding tissue. Damage could occur at the stalk end due to wrenching when the banana bunch is pulled of the stem, at the bottom end when the bunch hits the container, and anywhere else the bananas get bumped in transit. If you're getting bananas with brown on them, consider yourself lucky, the only bananas I ever see are very far from ripe - thank you very much, modern commercial food distribution system. [[User:Franamax|Franamax]] ([[User talk:Franamax|talk]]) 03:33, 8 October 2008 (UTC)

: My dad used to bite out the bruised bits of bananas before slicing them onto his breakfast cereal. (I don't know whether he still eats breakfast cereal.) The texture may be icky to some, but it won't hurt you, if that's what you're asking. —[[User:Tamfang|Tamfang]] ([[User talk:Tamfang|talk]]) 06:44, 8 October 2008 (UTC)

== Nobel Prize winners ==

Have there been any Nobel Prize winners whose work has since been completely discredited or otherwise found to be useless/incorrect? -[[User:Elmer Clark|Elmer Clark]] ([[User talk:Elmer Clark|talk]]) 20:41, 7 October 2008 (UTC)
:Interesting question! Since you posted in Science, I assume you're less interested in the [[Nobel Peace Prize]], where [[Rigoberta_Mench%C3%BA]] and [[Henry Kissinger]] come to mind. --[[User:TotoBaggins|Sean]] 21:45, 7 October 2008 (UTC)

::If this were about Nobel laureates generally, I'd mention [[Paul Johann Ludwig von Heyse|Paul Heyse]], who won the 1910 Literature prize. One of the judges said "Germany has not had a greater literary genius since [[Johann Wolfgang von Goethe|Goethe]]" - but history has been rather less kind to Heyse. -- [[User:JackofOz|JackofOz]] ([[User talk:JackofOz|talk]]) 01:12, 8 October 2008 (UTC)

:Off the top of my head, the closest I could come up with is [[Neils Bohr]], who won the [[Nobel Prize in Physics]] for devising the [[Bohr model]] of the atom. Its not that the Bohr model was wrong, in fact it is perfectly accurate for any two particle system. The problem is, in practical terms it means it predicts the electronic structure of the Hydrogen atom (which has an electron and the nucleus) and ''nothing else''. The Bohr model breaks down for any atom with more than one electron in it; so while it works for the [[Helium|He<sup>+1</sup>]] ion, it doesn't work for the He atom. It was an important step in the modern understanding of the electronic structure of the atom, but other more recent models, such as the [[Quantum chemistry|wave mechanical model]] of the atom as derived by [[Erwin Schrödinger]], [[Paul Dirac]], [[Louis de Broglie]], and others is far more useful, and as a model it contains the Bohr model as a special case. Still, Bohr is important as a key step in our understanding of the atom, and his contributions should not be deminished merely because better models came along later. --[[User:Jayron32|Jayron32]].[[User talk:Jayron32|<small>talk</small>]].[[Special:Contributions/Jayron32|<small>contribs</small>]] 21:37, 7 October 2008 (UTC)

::Bohr's idea of quantization of angular momentum also inspired [[Louis de Broglie|de Broglie]] whose's work latter inspired Erwin Schrödinger so that he came up with [[Schrödinger equation]]. I believe that the modern quantum theory couldn't exist without Bohr's breakthrough. - [[User:Justin545|Justin545]] ([[User talk:Justin545|talk]]) 00:35, 8 October 2008 (UTC)
:::[[Johannes Andreas Grib Fibiger]] is an interesting case, especially in light of this week's award to [[Harald zur Hausen]]. --[[User:Arcadian|Arcadian]] ([[User talk:Arcadian|talk]]) 09:47, 8 October 2008 (UTC)

:Argh - the guy's name escapes me for the moment. There is at least one case where someone made a seemingly great discovery - and it was disproven shortly before the Nobel was awarded - so they gave him the prize anyway but changed the citation from that very specific discovery to some kind of vague lifetime-achievement award...but the guy's name escapes me right now. [[User:SteveBaker|SteveBaker]] ([[User talk:SteveBaker|talk]]) 13:55, 8 October 2008 (UTC)

:In 1912 the swedes awarded the physics prize to one of their own, for the invention of an automatic lighthouse. Useful but trivial on the nobel scale. In 1949 the Nobel Prize in Medicine was awarded for an advance in [[Lobotomy]]. [[User:EverGreg|EverGreg]] ([[User talk:EverGreg|talk]]) 20:48, 8 October 2008 (UTC)
::Times sure do change and science advances. ''Now'', that 1949 choice seems unlikely, but at the time it was a no-brainer. [[User:DMacks|DMacks]] ([[User talk:DMacks|talk]]) 21:05, 8 October 2008 (UTC)
:::<small>Ba-da-bomp! And the Nobel Prize for today's best joke goes to...DMacks. :) [[User:Franamax|Franamax]] ([[User talk:Franamax|talk]]) 22:00, 8 October 2008 (UTC)</small>

Interesting answers, thanks - Fibiger at the very least certainly seems to qualify. -[[User:Elmer Clark|Elmer Clark]] ([[User talk:Elmer Clark|talk]]) 12:44, 9 October 2008 (UTC)

== honey locust spines: poisonous? ==

I just tore an enormous (23 centimeter long) spine off of a [[honey locust]] near here and in the process my left hand got jabbed. It didn't break the skin or draw any blood, but there's a small, red raised bump there like an insect bite that itches a little and the area around it is somewhat red and feels sore. I heard that honey locusts have some sort of toxin in their thorns that can be dangerous, but I can't find any information on it at all. Is this true? If so, do I need to seek medical attention? Can this get infected?

Any help is appreciated. Thanks.
[[Special:Contributions/63.245.144.77|63.245.144.77]] ([[User talk:63.245.144.77|talk]]) 20:44, 7 October 2008 (UTC)

:Sorry, we can't give medical advice. Please contact your doctor's office. The nurses there will be able to tell you what sort of medical attention (if any) you need. Also, note that some locations have a [[poison control]] telephone number - they are usually staffed with medically trained people, and are very helpful with "is this plant poisonous" type questions. -- [[Special:Contributions/128.104.112.147|128.104.112.147]] ([[User talk:128.104.112.147|talk]]) 23:39, 7 October 2008 (UTC)

== Holt Biology ==

What is a phospholipid layer that covers a cell`s surface and acts as a barrier between the inside of a cell and the cell`s environment? <span style="font-size: smallest;" class="autosigned">—Preceding [[Wikipedia:Signatures|unsigned]] comment added by [[Special:Contributions/75.68.106.206|75.68.106.206]] ([[User talk:75.68.106.206|talk]]) 21:14, 7 October 2008 (UTC)</span><!-- Template:UnsignedIP --> <!--Autosigned by SineBot-->

:Your Holt Biology book should have the answer. Please post all your homework questions at once so we don't have to repeat multiple times that we are not here to do your homework. -- [[User:Kainaw|<font color='#ff0000'>k</font><font color='#cc0033'>a</font><font color='#990066'>i</font><font color='#660099'>n</font><font color='#3300cc'>a</font><font color='#0000ff'>w</font>]][[User talk:Kainaw|&trade;]] 21:22, 7 October 2008 (UTC)

::See comments below on the plant cells. Be aware that while Wikipedia has an article on [[Cell (biology)|Cells]], which I recommend that you read, your Holt Biology book is likely much easier to follow, and if you read it, it will give you the answer. --[[User:Jayron32|Jayron32]].[[User talk:Jayron32|<small>talk</small>]].[[Special:Contributions/Jayron32|<small>contribs</small>]] 21:23, 7 October 2008 (UTC)

==Hearing==

Hi all,

Hypothetically, and I do mean hypothetically because I’m not asking for medical advice, how does one increase the results of a hearing test (only a temporary basis, not permanently).


I’ve heard that:
1. One’s hearing is better in the morning (so take the hearing test in the morning) – there will be a slight benefit.
2. Wear headphones (like the headphones for ear protection with firearms) prior to the hearing test. This will allow one to “rest” the ear drums so they’ll be more “sensitive” during the hearing test.

Any other thoughts (again, not seeking medical advice).
Thx!
[[User:Rangermike|Rangermike]] ([[User talk:Rangermike|talk]]) 21:15, 7 October 2008 (UTC)

:I had an uncle (until my aunt divorced him) who was practically deaf from working on jet aircraft. He needed to pass a hearing test to stay in the Navy Reserve or National Guard (don't remember which). What he did was wear ear plugs for a week or two before the test. Then, he would barely pass. He never tried it without the earplugs before the test, so I can't make a claim that it actually helped. -- [[User:Kainaw|<font color='#ff0000'>k</font><font color='#cc0033'>a</font><font color='#990066'>i</font><font color='#660099'>n</font><font color='#3300cc'>a</font><font color='#0000ff'>w</font>]][[User talk:Kainaw|&trade;]] 23:39, 7 October 2008 (UTC)
:::Fully off-topic - what is that guy called now? Ex-uncle? Uncle infinitely-removed? Former uncle (is this one best)? And for that matter, how do I refer to events that occured when my ex-wife was still just my girlfriend? SciRefDesk is probably not the best place to ask, I suppose... [[User:Franamax|Franamax]] ([[User talk:Franamax|talk]]) 05:45, 8 October 2008 (UTC)

::::Um - he is called "the ass who tried to stay in the reserves so he could spend one weekend a month in another city where he married another woman and, eventually, couldn't keep the two wives apart, got caught, and got dumped by both of them", or "the ass" for short. -- [[User:Kainaw|<font color='#ff0000'>k</font><font color='#cc0033'>a</font><font color='#990066'>i</font><font color='#660099'>n</font><font color='#3300cc'>a</font><font color='#0000ff'>w</font>]][[User talk:Kainaw|&trade;]] 13:16, 8 October 2008 (UTC)

:::::Wait, seriously? [[User:Plasticup|<b><font color="#0080FF">Plasticup</font></b>]] [[User_Talk:Plasticup |<font color="#2A8E82"><sup><small>T</small></sup></font>]]/[[Special:Contributions/Plasticup|<font color="#2A8E82"><small>C</small></font>]] 04:20, 10 October 2008 (UTC)
::::::I was thinking that too - good God man, ''two'' wives? I had enough trouble with just the one. OTOH man in unhappy marriage meets woman fresh out of unhappy marriage - Franamax results and mummy/daddy live happily together 'til death do them part. I suppose that "the ass" is always a relative term (though Kainaw seems to have identified the type specimen). [[User:Franamax|Franamax]] ([[User talk:Franamax|talk]]) 07:25, 10 October 2008 (UTC)

== Holt Biology ==

what is an organelle found in plant and algae cells where photosynthesis occurs? <span style="font-size: smallest;" class="autosigned">—Preceding [[Wikipedia:Signatures|unsigned]] comment added by [[Special:Contributions/75.68.106.206|75.68.106.206]] ([[User talk:75.68.106.206|talk]]) 21:17, 7 October 2008 (UTC)</span><!-- Template:UnsignedIP --> <!--Autosigned by SineBot-->
:You know, if you read your Holt Biology book, you may find the answer very easily. I bet you wouldn't have to look for more than 2 or 3 minutes. As an aside, you should also read the disclaimer at the top of this page, where it mentions that we aren't here to answer your homework questions. If reading your textbook is too much, you may want to read the Wikipedia article on [[Plant cells]]. Toodles. --[[User:Jayron32|Jayron32]].[[User talk:Jayron32|<small>talk</small>]].[[Special:Contributions/Jayron32|<small>contribs</small>]] 21:20, 7 October 2008 (UTC)
::Really you'd get a much faster answer by typing [[photosynthesis]] into the search box than we could ever provide, I just don't get it. <small>-- [[User:Mad031683|<font face="Courier" color="Green">Mad031683</font>]] ([[User talk:Mad031683|talk]])</small> 22:07, 7 October 2008 (UTC)
:::What is, "I'll take [[chloroplast]] for 200, Alex"? - <span style="font-family: cursive">[[User:Nunh-huh|Nunh-huh]]</span> 06:47, 8 October 2008 (UTC)
:::: An acceptable phrasing for choosing a 200 dollar question on the subject of chloroplasts on the show ''Jeopardy''? <span style="font-size: smallest;" class="autosigned">—Preceding [[Wikipedia:Signatures|unsigned]] comment added by [[Special:Contributions/82.120.232.170|82.120.232.170]] ([[User talk:82.120.232.170|talk]]) 19:56, 8 October 2008 (UTC)</span><!-- Template:UnsignedIP --> <!--Autosigned by SineBot-->
: And always remember, google is your friend. [[User:Sjschen|Sjschen]] ([[User talk:Sjschen|talk]]) 03:51, 9 October 2008 (UTC)

== steam rockets ==

I am interested in steam rockets. What can you tell me about them? I specifically wanted to know about heating the steam. The small steam rockets I've seen are preheated with a torch and then released. Is it possible to have an onboard heating system on a relatively small rocket? What would that be? Any additional information would be very helpful. Thank you.[[User:Wizardh2o|Wizardh2o]] ([[User talk:Wizardh2o|talk]]) 22:43, 7 October 2008 (UTC)

:We have an article on [[hot water rocket]]s. While it's possible to do an onboard heating device, it's inefficient -- you're burning fuel to heat water not just to "steam" but to "very hot steam" (to ensure sufficient pressure) when you could just be burning fuel for the pressure of the initial combustion and leaving the weight of the water out. So while it's possible, it's going to be rare. &mdash; [[User talk:Lomn|Lomn]] 23:09, 7 October 2008 (UTC)

:: One exception occurs in science-fiction: fusion reactors produce heat, but nothing much you could expell directly, so the writer imagines a tiny super-storng nozzle through TINY amounts of insanely super-heated water are expelled. (This is a space ship). Can't remember who the writer was, someone famous like Asimov... <span style="font-size: smaller;" class="autosigned">—Preceding [[Wikipedia:Signatures|unsigned]] comment added by [[Special:Contributions/82.120.232.170|82.120.232.170]] ([[User talk:82.120.232.170|talk]]) 18:00, 8 October 2008 (UTC)</span><!-- Template:UnsignedIP --> <!--Autosigned by SineBot-->

:::This is the basic idea behind the [[nuclear thermal rocket]], except that for an NTR, Very Bad Things can happen if you expel the fuel. --[[User:Carnildo|Carnildo]] ([[User talk:Carnildo|talk]]) 21:50, 8 October 2008 (UTC)

::::I think you mean [[Nuclear salt-water rocket]], and the rocket doesn't expell fuel, it expells propellent (aka reaction mass). [[Special:Contributions/98.221.85.188|98.221.85.188]] ([[User talk:98.221.85.188|talk]]) 04:21, 9 October 2008
:::::I meant nuclear thermal. A nuclear salt-water rocket *does* expel the fuel -- that's what the uranium salts in the water are. --[[User:Carnildo|Carnildo]] ([[User talk:Carnildo|talk]]) 20:05, 9 October 2008 (UTC)

::::::No, you meant Nuclear salt-water. And what the salt-water rocket expells is reaction mass. Sometimes fuel can be used as reaction mass, but if it's being expelled, it's called reaction mass, not fuel. [[Special:Contributions/98.221.85.188|98.221.85.188]] ([[User talk:98.221.85.188|talk]]) 14:44, 10 October 2008 (UTC)

:::::::No, I mean "nuclear thermal". As in a solid-state nuclear reactor ("nuclear") that heats ("thermal") water, expelling it out the back of the rocket at high velocity. --[[User:Carnildo|Carnildo]] ([[User talk:Carnildo|talk]]) 22:17, 10 October 2008 (UTC)

::::::::Isn't that what the ip is talking about? You mentioned fuel being expelled as uranium salts in the water, so presumably, you indeed are talking about what he is saying. [[User:ScienceApe|ScienceApe]] ([[User talk:ScienceApe|talk]]) 00:12, 11 October 2008 (UTC)

:::What about those new racing motorcycles that have rockets? they must have on board heating. How do they work? And what about those mini helicopters with rockets on their blades? Or that car that went 260 mph for a quarter mile? It had rockets also, that I'm pretty sure were steam. I really could use the information. [[Special:Contributions/74.71.203.132|74.71.203.132]] ([[User talk:74.71.203.132|talk]]) 17:54, 9 October 2008 (UTC)

== Putting car in park ==

Is it a good idea to shift my car (a 2004 automatic) into park when I'm at a long light or waiting for a left turn? Can doing this alot damage my car? Does it save any gas?[[Special:Contributions/97.118.170.250|97.118.170.250]] ([[User talk:97.118.170.250|talk]]) 23:08, 7 October 2008 (UTC)
:For an automatic? It's not likely to affect the car negatively, though in many cases it'll burn ''more'' gas -- my auto, and most I've driven, idle faster in park/neutral than in drive. There's an advantage to shifting to neutral in a stick shift so as to reduce wear on the clutch. &mdash; [[User talk:Lomn|Lomn]] 23:11, 7 October 2008 (UTC)
::Is there much wear on a clutch when it's fully disengaged? --[[User:Tango|Tango]] ([[User talk:Tango|talk]]) 23:19, 7 October 2008 (UTC)
::::According to the Car Talk guys, keeping the clutch pedal fully in causes wear on the [http://www.cartalk.com/content/columns/Archive/1993/April/10.html throw-out bearing]. They recommend shifting to neutral. &mdash; [[User talk:Lomn|Lomn]] 00:02, 8 October 2008 (UTC)

:::The only advantage of shifting to park that I can think of is to avoid putting your foot on the brake. The disadvantage is that you'll need to ensure you shift into drive (not neutral and definitely not reverse) when the light turns green. As for the transmission (it's automatic, so you aren't manually messing with the clutch), each car I've owned has run just fine for at least 10 years without transmission problems. From my experience, people I know with automatic transmission problems are those who gun it and race from light to light - which has nothing to do with sitting at a light. My sister-in-law has two foot settings: all the way down or all the way up. She goes through a transmission a year. -- [[User:Kainaw|<font color='#ff0000'>k</font><font color='#cc0033'>a</font><font color='#990066'>i</font><font color='#660099'>n</font><font color='#3300cc'>a</font><font color='#0000ff'>w</font>]][[User talk:Kainaw|&trade;]] 23:36, 7 October 2008 (UTC)

But will it save me any gas?[[Special:Contributions/97.118.170.250|97.118.170.250]] ([[User talk:97.118.170.250|talk]]) 23:41, 7 October 2008 (UTC)

:Did you read Lomn's response above? He states that his experience is that it will burn ''more'' gas, not less. -- [[User:Kainaw|<font color='#ff0000'>k</font><font color='#cc0033'>a</font><font color='#990066'>i</font><font color='#660099'>n</font><font color='#3300cc'>a</font><font color='#0000ff'>w</font>]][[User talk:Kainaw|&trade;]] 23:45, 7 October 2008 (UTC)

::Re Lomm's observation: isn't the idling speedup due to removing the transmission load from the engine rather than increasing the fuel supply? -[[User:Hydnjo|hydnjo]] [[User talk:Hydnjo|talk]] 00:40, 8 October 2008 (UTC)

:::It is difficult to say in modern vehicles where a computer controls the fuel supply. In the good old days, shortly after we had to use our feet to make the car move, the gas pedal actually increased/decreased the amount of fuel to the engine. So, regardless of what else was happening, not having your foot on the pedal would make it consume fuel at the idling rate at all times. The OP said this was a 2004 car, so it is one of those newfangled computer-controlled ones. -- [[User:Kainaw|<font color='#ff0000'>k</font><font color='#cc0033'>a</font><font color='#990066'>i</font><font color='#660099'>n</font><font color='#3300cc'>a</font><font color='#0000ff'>w</font>]][[User talk:Kainaw|&trade;]] 01:53, 8 October 2008 (UTC)
:In re the above:
:*Switching an automatic transmission into Park should alert the computer to select the most economical fuel mode possible. At certain times, it could select "curb idle", which is a higher idle rate to maintain alternator output. If you aren't running your A/C, there's no particular reason I can think of that the computer would choose to send ''more'' fuel to the engine. However, given the same amount of fuel, shifting to Park would stop your motor from pumping liquid around the transmission torque converter, so I'd expect the engine to rev a little higher without that load.
:*And for a manual transmission, shifting to neutral is always good, not least because your leg doesn't get tired. However, I think Lomn's Car Guys link is a piece of semi-crap. Their piece ends with an appeal to send them $3 to find out the rest of the mysteries, which is a little suspicious. Keeping your clutch depressed means there is a constant load on the throw-out bearing, but there is no other load - the bearing is not rotating, it's just sitting there with the incredibly hard steel of the bearing-balls and bearing-races subject to the pressure of - your foot! Contrast keeping the clutch fully disengaged though with [[Clutch_control#Riding_the_clutch|riding the clutch]], or keeping your foot lazily off the floor, so that the throw-out bearing is under pedal pressure ''and'' is rotating the partially-engaged clutch - that's definitely not good! But I've gotten 14 years and 440,001 km out of a single clutch (1992 VW Golf), and I never worried that much about shifting to neutral. The Car Guys might be right though, maybe the clutch would have blown at 600,000 km... :) [[User:Franamax|Franamax]] ([[User talk:Franamax|talk]]) 04:08, 8 October 2008 (UTC)

::Keeping your foot on the clutch in a stick-shift car does indeed wear out that bearing prematurely (it's sometimes called the "thrust washer"...same thing). But in a modern car, with a typical driver, it's pretty unlikely that that'll wear out before the clutch itself - so while it's a good idea to shift into neutral and take your foot off the clutch when stopped - it's not a really critical thing. On an automatic, it's irrelevant because there is a fluid coupling clutch. There are many cases though where it's '''DANGEROUS''' to leave the car in gear with your foot on the clutch - for example, if you are stopped at a junction and you get rear-ended, your foot will almost certainly slip off of the clutch and you'll be launched forward into oncoming traffic - turning a minor fender-bender into a life-threatening situation. If you are in neutral with your foot on the brake (either in an automatic or a manual, then if your foot should happen to slip (eg as a result of a rear-ending or wet shoes or something) then at least you'll only roll rather gently forwards. Best of all is to put the car in neutral and put on the parking brake. In an automatic, putting it in park should have the same effect - but I have heard that the pin that slips into the transmission when you put the car in park can get worn out from excessive use...so I'm still going to say "Neutral with the parking brake on" is the best option and "In park" as the second best. I would be very surprised indeed to discover any difference whatever in gas consumption with any of the above techniques so long as you aren't revving your engine to keep your turbo spooled up in preparation for an impending burn-out when the light turns green!
:::Rather the contrary, if you're sitting in (manual transmission) neutral and get rear-ended, you ''will'' be launched into the intersection if your foot slips off the brake. If you're in gear with the clutch down, when your foot slips off the car will stall and engine braking will ''prevent'' you from being shoved into the intersection. Neutral ''with'' parking brake on would be best though. [[User:Franamax|Franamax]] ([[User talk:Franamax|talk]]) 21:31, 8 October 2008 (UTC)
::::I disagree about the stalling thing. I know of one person who had this happen - the car didn't stall and she ended up being side-swiped by a Jeep that was going about 40 to 50mph - which rolled the Jeep - totalled her car and might have killed her had it not been for her side-impact airbag and door reinforcement. If you think about it, that's not surprising. The engine stalls when you are at a dead stop and dump the clutch too suddenly - but if you've just been propelled forwards by the force of a rear-ending, the RPM may well be efficiently matched to the speed of the wheels when the clutch bites - and far from stalling - the car may actually accellerate into the intersection. A lot will depend on the car - my MINI Cooper'S is pretty hard to stall like that - it has a pretty flat torque curve so you tend to get a bit of tyre squeak and then it's perfectly happy. Personally - I don't recommend taking that bet. If that's your driving style - you should change it ASAP. In neutral with the parking brake on - or ''possibly'' in gear with the clutch down AND the parking brake on - are the only safe possibilities with a stickshift car. [[User:SteveBaker|SteveBaker]] ([[User talk:SteveBaker|talk]]) 02:14, 9 October 2008 (UTC)
:::::Steve, I quail at the thought of ever disagreeing with you, what with you being pretty much completely always right and all (I mean that!) - but I have to disagree here. Yes of course using the parking brake is always the best solution, and by that we probably both mean the ''handbrake'' available to hand in a small car. Even then, most people will not engage the handbrake at a common driving stop. Consider now a traffic stop where the parking brake is not engaged, and the only variable is whether or not the manual transmission is engaged in gear, or in neutral (the driver's feet disengage from all pedals on impact). In the neutral case, the car will accept the momemtum transferred on impact (minus energy of deformation) and (almost) immediately adopt a forward velocity. In the case where the transmission is engaged, regardless of whether or not the engine stalls, the rotating/translating mass of the engine will participate in the collision energetics. Unless you were sitting there revving the engine, including the inertial moment of the engine elements into the energy equation can ''only'' slow the car down. Remember that you are not feeding extra fuel to the engine - whacking the car from behind may cause the auto with an engaged gear to move slowly forward making a "putt-putt" sound from the idling engine, but I see no possible way it would cause sufficient velocity to participate in an intersection crash. Smacking a car in neutral gear though - think about billiard balls, and the pocket is on the other side of the intersection.
:::::As far as changing my driving style, I'll let you know when I've completed ny <u>second</u> million kilometres, the first has gone pretty well. (FD - 2 accidents - early 20's [age, not year!] - one, making a U-turn without looking back, the other looking for something in my back seat) I've in the past read advice about sitting still in a manual auto, it's uniformly to stay in gear with the clutch down. You are absolutely right that the parking brake is the way to go, but in abeyance of that advice - keep it in gear. [[User:Franamax|Franamax]] ([[User talk:Franamax|talk]]) 03:28, 9 October 2008 (UTC)
:::::::<small>Actually, I'm not sure - there may be no pockets in billiards. So instead, think of it as a snooker ball or a pool ball. I know how to play and lose at both of those. :) [[User:Franamax|Franamax]] ([[User talk:Franamax|talk]]) 03:33, 9 October 2008 (UTC) </small>
::::::When the engine is idling, there is fuel going into it. You put a car in gear and take your feet of all the pedals it will move forwards (assuming it doesn't stall, and the fact that it's already moving slightly from the impact will help there as Steve says). I'm not sure the velocity imparted by the impact would be enough to prevent the car stalling, but it could well be - remember the engine is disengaged at the time of the impact (since your foot is on the clutch) so there is no engine braking in that first fraction of a second. I guess it all depends on how quickly your foot falls off the clutch pedal. That said, I don't think I know anyone that puts their car in neutral every time they stop at lights, and my driving instructor never told me to. Of course, if you have your feet on both the clutch and the foot brake, you should be fine - the brake will stop you moving forwards from the impact even if your foot does then fall off it, so then the engine will stall when you lift the clutch. --[[User:Tango|Tango]] ([[User talk:Tango|talk]]) 20:50, 9 October 2008 (UTC)
:::::::Just think about the dynamics. You've been rear-ended and your feet have been knocked off all the pedals. If you're in neutral, your car is free to move forward with all the imparted momentum. If the car is in gear, part of the momentum is dissipated into accelerating the engine. The idle fuel feed is immaterial - how fast does your car go when your foot's not on the gas pedal? Chugging forward into the intersection is not the problem, getting shoved forward is. If you get rear-ended hard enough to knock your feet off the pedals, you want every possible means to stay out of that intersection.
:::::::If I take my car out of gear whilst sitting on the active road area, I pull the handbrake - always. But usually I hold the clutch in, with the other foot on the brake pedal. I also do the left-turn thing where you pull out and crank the wheel left to get a good sight-line, then turn the wheel back so you don't get rammed into oncoming traffic. ''And'' the left-turn thing where you hang back until at least one other car is behind you, then move forward a bit. There's also the best practice thing where you stop 20 feet behind the next car at a stop-light, wait 'til at least one other car stops behind you, then move up - I don't do that though, since I've paid for an airbag I didn't ask for, meant to save the life of an overweight American male who isn't using his seatbelt. :) I actually spend most of my driving time evaluating threats - and enjoying my machine. [[User:Franamax|Franamax]] ([[User talk:Franamax|talk]]) 07:17, 10 October 2008 (UTC)
::::::::I've never actually measured the speed a car goes when idling, but I heard 8mph given as a figure once, which is plenty fast enough to cause a problem at a junction. --[[User:Tango|Tango]] ([[User talk:Tango|talk]]) 21:15, 10 October 2008 (UTC)
:::::::::And if you're sitting in neutral and get rear-ended by a car moving at 30 mph, what is your resulting velocity? More or less than 8 mph? [[User:Franamax|Franamax]] ([[User talk:Franamax|talk]]) 01:06, 11 October 2008 (UTC)

= October 8 =

== Headset to hear dictations ==

What are the technical specifications that we have to look in a head phone to get maximum efficiency to hear dictations and comfort for wearing long hours?

:As far as comfort goes, that's not likely to be a question of technical specifications. Your head and ears are pretty unique in their shape, and what is comfortable for one person may not be comfortable for you, and vice versa. -- [[User:Captain Disdain|Captain Disdain]] ([[User talk:Captain Disdain|talk]]) 01:17, 8 October 2008 (UTC)

:: For example, I switched from earmuffs (which I otherwise prefer) to plugs after I got new glasses whose shafts stand out further from my head than before. —[[User:Tamfang|Tamfang]] ([[User talk:Tamfang|talk]]) 05:34, 8 October 2008 (UTC)

== Sensory Integration Disorder and its Impact on Potty Training ==

I have a 4 yr old daughter who has SID and is very hypersensitive. She has been working with OT and PT since she was 5 mths old. No one has any clear information to give me on how to potty train! The developmental specialist states to give her time- the uroligists states she must get it done b/c of the severe negative side effects it has had on her (kidney infections that have led to hospitalizations). She is very fearful of even sitting on the toilet much less wanting to be in the bathroom when it flushes! I have read all I can find on SID and own numerous books on potty training. Are there methods used for children with SID to help with this issue??

Thank you- <span style="font-size: smallest;" class="autosigned">—Preceding [[Wikipedia:Signatures|unsigned]] comment added by [[Special:Contributions/12.202.108.191|12.202.108.191]] ([[User talk:12.202.108.191|talk]]) 03:25, 8 October 2008 (UTC)</span><!-- Template:UnsignedIP --> <!--Autosigned by SineBot-->
:Have you asked the medical professionals for locations of appropriate support groups? Have you tried searching on the web for support groups that address this specific issue? There are lots of parents here, but we're really not allowed to give medical advice on specific issues. [[User:Franamax|Franamax]] ([[User talk:Franamax|talk]]) 05:01, 8 October 2008 (UTC)
::And there's no particular reason the child ''must'' be in the bathroom when the toilet flushes. She can be standing just outside the door watching when you flush the toilet, and then she can hold a long stick to help you flush the toilet, and then she can use the stick herself. But that's starting to be medical advice... [[User:Franamax|Franamax]] ([[User talk:Franamax|talk]]) 05:07, 8 October 2008 (UTC)

== Strange gender inequality in the dental industry ==

I've been to numerous dentists. Why are the dentists always only men and the assistants always only women? [[User:Are you ready for IPv6?|Are you ready for IPv6?]] ([[User talk:Are you ready for IPv6?|talk]]) 03:49, 8 October 2008 (UTC)
:That's an interesting question. Have you tried to seek out dental clinics where the dentists are female? As far as the gender split goes:
:*Dentist does seem to be a male-dominated field. A discussion on possible gender-bias can be found [http://www.nature.com/bdj/journal/v188/n1/full/4800372a.html here]. Stand-alone professional fields such as dentistry would seem good ground for women to avoid gender bias, since they have only to fulfill the qualifications. I would speculate that those women so inclined might gravitate instead to pharmacology or in particular physiology (M.D.-type doctor rather than D.D.S-doctor). Perhaps those women inclined to a medical field either go big or go home (''pace'' to any dentists who might think I denigrate their important field - but it does rank just a little under M.D.).
:*As far as dental assistants or [[dental hygienist]]s, I would speculate that the field offers a good way for women of child-bearing age to acquire a valuable transferable skill that offers them maximum flexibility as to hours worked, balanced with a good income. Also, women are probably just better as dental assistants, since they're more empathetic, interact naturally with children, etc. You can read that as a sexist statement, but I mean it as a statement of great admiration.
:*And the factor of bearing and caring for children must always be considered in choice of training and work for women - they're the ones making them after all. :) In the case you question though, the balance does seem somewhat skewed. [[User:Franamax|Franamax]] ([[User talk:Franamax|talk]]) 04:55, 8 October 2008 (UTC)

:My current dentist is female, my former dentist was female. Of the 4 dentists over my life-time 2 have been female. Of course that's my perspective and so statistics might not back this up. Just throwing that in there. [[Special:Contributions/194.221.133.226|194.221.133.226]] ([[User talk:194.221.133.226|talk]]) 10:32, 8 October 2008 (UTC)

:Could it be the case that greater (on average) physical strength and large hands are an advantage when pulling teeth? Or that the general wisdom is such - thus discouraging some women from taking up dentistry? A personal observation - the only time a female dentist tried to remove one of my teeth (I've had a lot of teeth removed), she wasn't strong enough to budge it and had to go fetch a male colleague. --[[User:Kurt Shaped Box|Kurt Shaped Box]] ([[User talk:Kurt Shaped Box|talk]]) 10:59, 8 October 2008 (UTC)
::Personal observation #2 - my lower wisdom teeth had twisted roots and the (male) dentist couldn't pull them no-how. He ended up jack-hammering them into pieces (which I still have). As well as the pain from the extraction site, I had a bruise on my jaw from where he braced his hand trying to pull. The point about physical strength may have some merit. [[User:Franamax|Franamax]] ([[User talk:Franamax|talk]]) 21:24, 8 October 2008 (UTC)

:[http://www.spiritofcaring.com/public/488.cfm This site] has some numbers. It says there is an 83%/17% male/female split among all private practice dentists in the US. It's interesting to note that at a prestigious veterinarian school in my area there is a similar split in the other direction. --[[User:TotoBaggins|Sean]] 14:51, 8 October 2008 (UTC)

:It's probably the same reason that most Doctors (traditionally) have been male, and most Nurses female. "Dentist [[analogy|is to]] Doctor as [[Dental hygienist]] is to Nurse" (FWIW, my current dentist is female, and I know of male hygienists, just like I know female doctors and male nurses) -- [[Special:Contributions/128.104.112.147|128.104.112.147]] ([[User talk:128.104.112.147|talk]]) 15:40, 8 October 2008 (UTC)
::A [[dental hygienist]] is different from a [[dental nurse]]. --[[User:Tango|Tango]] ([[User talk:Tango|talk]]) 23:23, 8 October 2008 (UTC)



I'm a male and I don't work out very well and probably every woman in the miitary is stronger than me so I don't think it's strength. As for pregnancy, dentists often share a practice with another dentist and dentists themselves generally see the patient for two minutes during checkup. I also have noticed there's a lot of female vets. I'm not really quite sure how to really search for lots of dentists. I usually call 800-DENTIST when I move, but my last move within about 50 miles only one dentist was registered. That dentist had a nice office with fancy machines so I thought he'd be good. It turns out that he pays for it all by not just charging 4 times more for an exam, but will give false diagnosis and tell people they need expensive things done that only harm their teeth. I wasn't certain until I searched on the internet about this. There's countless cases of people who moved to a new location, got a new dentist, and despite having no cavities or other problems for 10-15+ years their new dentist claims to find 5-12 cavities that don't exist just so he can mutilate a person's body to make extra money. It's rather common. I take it dentists don't have to take the hipocratic oath. [[User:Are you ready for IPv6?|Are you ready for IPv6?]] ([[User talk:Are you ready for IPv6?|talk]]) 01:39, 9 October 2008 (UTC)
:<s>Well, if you just wanted to rant about how evil dentists are, you could have saved us some time and effort answering what looked like a legitimate question. How's this then? Dentists are a male-dominated caste placed on this earth solely to suck away our cash and destroy our bodies. There you go, you read it here on Wikipedia, so it must be true.</s> Can we close this thread now? [[User:Franamax|Franamax]] ([[User talk:Franamax|talk]]) 02:51, 9 October 2008 (UTC)
::That wasn't reason for the question. I just bought it up along with a lot of othe things because I saw a lot of people discussing this. [[User:Are you ready for IPv6?|Are you ready for IPv6?]] ([[User talk:Are you ready for IPv6?|talk]]) 03:41, 9 October 2008 (UTC)
:::OK, I'll retract any suggestion that you posed an improper question in the first place. However, the later post you made strays over the line into our striction that the RefDesk is not a forum for opinions, nor is it a soapbox to put forth your own opinions. If you have a ''specific'' follow-up question, we can consider it. [[User:Franamax|Franamax]] ([[User talk:Franamax|talk]]) 03:47, 9 October 2008 (UTC)

== immume system ==

types of immumoglobine <small><span class="autosigned">—Preceding [[Wikipedia:Signatures|unsigned]] comment added by [[User:Ksneyhaa|Ksneyhaa]] ([[User talk:Ksneyhaa|talk]] • [[Special:Contributions/Ksneyhaa|contribs]]) 06:37, 8 October 2008 (UTC)</span></small><!-- Template:Unsigned --> <!--Autosigned by SineBot-->

:WIth regard to the [[immune system]], we discuss [[immunoglobulin]]s [[IgA]], [[IgD]], [[IgE]], [[IgG]], and [[IgM]]. - <span style="font-family: cursive">[[User:Nunh-huh|Nunh-huh]]</span> 06:43, 8 October 2008 (UTC)

== chemistry ==

two flasks A & B have equal volumes .flask A contains Hydrogen maintained at 300K while B contains methane gas maintained at 600K.
1.which flask contain greater number of molecules & hwmany times more <small><span class="autosigned">—Preceding [[Wikipedia:Signatures|unsigned]] comment added by [[User:Bhuwanntl|Bhuwanntl]] ([[User talk:Bhuwanntl|talk]] • [[Special:Contributions/Bhuwanntl|contribs]]) 07:55, 8 October 2008 (UTC)</span></small><!-- Template:Unsigned --> <!--Autosigned by SineBot-->
:"Do your own homework. The reference desk will not give you answers for your homework, although we will try to help you out if there is a specific part of your homework you do not understand. '''Make an effort to show that you have tried solving it first.'''" -- [[User:Aeluwas|Aeluwas]] ([[User talk:Aeluwas|talk]]) 08:53, 8 October 2008 (UTC)
:Your question is unanswerable, because you have not specified whether the two flasks have equal pressures. Take a look at [[combined gas laws]]. If we specify that pressure is equal, you're looking at [[Charles's law]]. - <span style="font-family: cursive">[[User:Nunh-huh|Nunh-huh]]</span> 09:00, 8 October 2008 (UTC)

== [[Aflatoxin]] in raw peeled peanuts ==

How can I know if raw peeled peanuts are contaminated by [[Aspergillus flavus]] and [[Aspergillus parasiticus]]? Is it possible to see it like [[mold]] on bread? [[User:Mr.K.|Mr.K.]] [[User_talk:Mr.K.|(talk)]] 12:14, 8 October 2008 (UTC)

:Peanuts in general are susceptible to aflatoxin contamination. You wouldn't see the aflatoxin itself (except under blacklight), but you might see the fungus that produces it. Cooking doesn't really affect it, since the toxins are heat stable (the fungus that produces them, however, is not). Fungus doesn't imply aflatoxin (not all fungi produce it), but lack of fungus doesn't guarantee safety. Like most toxins, a really small dose (i.e. one dubious peanut eaten per year) isn't going to kill you. If in doubt, don't eat it (no surprise there), but probably best to avoid the whole group if you see some fungus growing on anything in it. [[User:Somedumbyankee|SDY]] ([[User talk:Somedumbyankee|talk]]) 13:01, 8 October 2008 (UTC)

== Sleep injuries ==

I'm failing to find the medical term for injuries that occur while sleeping, such as pulling a muscle or separating a joint. Is there a term to classify those injuries? -- [[User:Kainaw|<font color='#ff0000'>k</font><font color='#cc0033'>a</font><font color='#990066'>i</font><font color='#660099'>n</font><font color='#3300cc'>a</font><font color='#0000ff'>w</font>]][[User talk:Kainaw|&trade;]] 13:18, 8 October 2008 (UTC)
::They don't happen often enough, and aren't intrinsically different from similar injuries occurring while awake, for there to be a common medical name for them. - <span style="font-family: cursive">[[User:Nunh-huh|Nunh-huh]]</span> 15:22, 8 October 2008 (UTC)

== Semen and Enzymes ==

Do the enzymes in washing detergent break down the proteins ect, that are in semen? i always wash my underwear seperately, because i worry that im simply spreading it all evenly on my clothes, and that microscopic amounts will rub off on where i sit ect. [[User:Zakbrak341|Zakbrak341]] ([[User talk:Zakbrak341|talk]]) 13:56, 8 October 2008 (UTC)
:If they're only microscopic amounts, what difference does it make? Also, why do you have so much semen in your underwear? You're meant to remove your clothes for that kind of thing... --[[User:Tango|Tango]] ([[User talk:Tango|talk]]) 14:10, 8 October 2008 (UTC)
:: Its not loads, just very small amounts of precum before i take them off ect, if got lots on them i'd just throw them away, but the thought of any amount of semen on anything bothers me, for a while i was doing a hot wash, so it would denature the protiens but then i worried that the heat would denature the enzymes in the powder, and if the enzymes break down the protiens into amino acids, then thats better than simply unfolding the protien into its secondary of primary structure, then i started worrying, what if these enzymes in the washing powder are specfic to protiens in common food stains only and have no effect on the protiens in semen, so now im just wash them in serveral changes on water with lots of detergent. <small><span class="autosigned">—Preceding [[Wikipedia:Signatures|unsigned]] comment added by [[User:Zakbrak341|Zakbrak341]] ([[User talk:Zakbrak341|talk]] • [[Special:Contributions/Zakbrak341|contribs]]) 14:31, 8 October 2008 (UTC)</span></small><!-- Template:Unsigned --> <!--Autosigned by SineBot-->

::Tango, two things. First, some people are kinda clean freaks when it comes to everything, so it's not really unusual to be concerned over whether semen is contaminating other clothes in the washing machine. Secondly, it's not really your place to ask him why he has semen in his underwear, you should only address his questions. [[Special:Contributions/98.221.85.188|98.221.85.188]] ([[User talk:98.221.85.188|talk]]) 14:35, 8 October 2008 (UTC)
:::Your detergent should [[emulsify]] fats, which means that it wraps little bubbles of detergent around your semen and stops it from sticking to any of your clothes. [[User:Plasticup|<b><font color="#0080FF">Plasticup</font></b>]] [[User_Talk:Plasticup |<font color="#2A8E82"><sup><small>T</small></sup></font>]]/[[Special:Contributions/Plasticup|<font color="#2A8E82"><small>C</small></font>]] 15:08, 8 October 2008 (UTC)
::::Except that semen is essentially fat-free. No emulsification. For the original questioner: the enzymes in washing detergents are designed to be as non-specific as possible in breaking down proteins, and should have no trouble with semen. - <span style="font-family: cursive">[[User:Nunh-huh|Nunh-huh]]</span> 15:19, 8 October 2008 (UTC)
::::Even if someone can get something which is soiled physically clean, meaning that the offending sunstance cannot be seen or even detected by CSI, they still remember it was once there. It can get more into [[Ritual purification|ritual purification]], like the concept of a ritual bath removing some iniquity or moral stain. I read a housewife's complaint to an advice column that her husband had soaked his sore toe in warm salt water (at the doctor's advice) in one of her expensive gourmet cooking pots, and that she had to throw the pot away because she would never afterward feel that it was "clean" regardless of scrubbing or dishwasher cleansing. If the questioner gets a reference to an authoritative laundering site which says a wash of some temperature on some cycle with certain cleaning products will render the garment as clean as it was when it came out of the package, then he might be saved the expense of frequently throwing garments away and buying replacements. Some people, even in this era of "cold water" energy-saving detergents wash a load of white cotton washcloths, towels, and underwear in the "hot" wash cycle with detergent and chlorine bleach, with an extra rinse, Not saying this is the authoritative answer to the original question, and there is also the question of enzymes versus bleach and hot versus cold water to eliminate biological stains. Interestingly there are loads of book references dealing with semen stains[[http://books.google.com/books?lr=&as_brr=0&q=%22semen+stain%22&btnG=Search+Books]] though few specifically address their removal. (Not to be confused with the mythical character "Seaman Staines" on the [[Captain Pugwash]] children's program). [[User:Edison|Edison]] ([[User talk:Edison|talk]]) 15:23, 8 October 2008 (UTC)

:You can check how good a job your selected detergent is doing by taking a look at your unmentionables under a black light before and after the wash. [[Special:Contributions/71.178.135.144|71.178.135.144]] ([[User talk:71.178.135.144|talk]]) 02:15, 9 October 2008 (UTC)
::Not a great method. While semen does fluroesce under a black light, it is not the only thing that does. Lots of detergents and fabric softeners do, as well. When you see, on crime shows, them use a black light to look for semen and/or blood, it's not the black light that proves its presence; the black light merely lets them know where the most likely places to test are. Its only after they have done a positive chemical test can they know that the stain that glows under black light is semen, or blood, or any of a number of other agents that also glow... --[[User:Jayron32|Jayron32]].[[User talk:Jayron32|<small>talk</small>]].[[Special:Contributions/Jayron32|<small>contribs</small>]] 02:24, 9 October 2008 (UTC)

== E=mc^2...Whatever that means. ==

Hey...what unit is the E in <math>E=mc^2</math>? Is it calories, Joules, tons, or what? I have been wondering about this for a while and saw the article on [[E=mc^2]] with its handy list of how much energy is contained in a dollar bill. It lists several units of measuring energy, which got me confused. When you convert mass to energy, is the mass converted to Joules or calories or another type of energy? [[User:10minnickm|31306D696E6E69636B6D]] ([[User talk:10minnickm|talk]]) 16:36, 8 October 2008 (UTC)

:Well, it defaults to kg·m<sup>2</sup>/s<sup>2</sup> (Joules), which it does say in the intro :)
::In the formula, c2 is the conversion factor required to convert from units of mass to units of energy. The formula does not depend on a specific system of units. In the International System of Units, the unit for energy is the joule, for mass the kilogram, and for speed meters per second. Note that 1 joule equals 1 . In unit-specific terms, E (in joules) = m (in kilograms) multiplied by (299,792,458 m/s)2.
:-- [[User:Macaddct1984|MacAddct1984]] <sup>([[User talk:Macaddct1984|talk]] &#149; [[Special:Contributions/Macaddct1984|contribs]])</sup> 16:42, 8 October 2008 (UTC)
::(ec) :Joules and calories are ''units of measurement'', not different "type[s] of energy". Just like you can measure speed as miles per hour or kilometers per hour or feet per second and can convert a speed from one to the other but it's still the "same speed". So whatever units you use for ''c'' and for ''m'' determines the units of ''E''. If that's not the unit you want, you can convert. Or you can choose what units of energy you want and then convert the mass and speed of light to use them. [[User:DMacks|DMacks]] ([[User talk:DMacks|talk]]) 16:43, 8 October 2008 (UTC)
:As long as you're consistent, you can use any unit system (for example, you could use [[erg]]s, but then you would have to measure the mass in grams and the speed of light in cm/s - see [[cgs]]). Usually scientists use [[SI unit]]s, where energy is measured in [[joule]]s. It's also common to see energy measured in [[electron volt]]s. --[[User:Tango|Tango]] ([[User talk:Tango|talk]]) 16:46, 8 October 2008 (UTC)
:: You're wrong: you can use any units that measure the same thing. For example, instead of asking how many 'joules' a certain number of 'kilograms' would result in according to e=mc^2, you can ask Google how many "horsepower-hours" (which measures the same thing joules do) a "pound" of stuff results in. Just google [http://www.google.com/search?hl=en&rls=en&hs=R2d&q=1+pound+%2A+c%5E2+in+horsepower%2Ahours&btnG=Search "1 pound * c^2 in horsepower*hours"]. You get "1 pound * (c^2) = 1.51859015 × 1010 horsepower * hours". In other words, if you had a magic reactor in your room that completely turned matter into energy, then with 1 pound of fuel you could output one horsepower (745 watts) for
15000000000000000000000000000000000000000000000000000000000000000000000000000000000000000000000000000 hours.

:::You haven't used the same equation, though, you've added a conversion factor (or, you've absorbed the conversion factor into the constant by measuring the speed of light in obscure units). You have to use consistent units, otherwise you quite clearly get nonsense. If I use kg for the mass and m/s for the speed, plug it in and I get joules for the energy, I can't just choose for that to be ergs or electron volts or horsepower hours, since that would be assuming they are all the same thing. To get to another unit I had to multiply by a conversion factor so then I'm actually using ''E''=''kmc''<sup>2</sup> where ''k'' conversion factor. --[[User:Tango|Tango]] ([[User talk:Tango|talk]]) 21:12, 8 October 2008 (UTC)
:::: No, you're just wrong. Take something simple: distance = speed * time. So, 60 miles per hour for two minutes = distance = 60 m/h * 2 minute = 120 minutes * m/h, ie "120 mile-per-hour minutes". And look [http://www.google.com/search?rls=en&q=120+mile+per+hour+*+minutes&sourceid=opera&ie=utf-8&oe=utf-8 120 mile per hour * minutes = 3.218688 kilometers] is a perfectly good answer. You can ask for it in feet, inches, anything. It's just a unit conversion, it's not solving an equation. <span style="font-size: smaller;" class="autosigned">—Preceding [[Wikipedia:Signatures|unsigned]] comment added by [[Special:Contributions/82.120.232.170|82.120.232.170]] ([[User talk:82.120.232.170|talk]]) 12:26, 11 October 2008 (UTC)</span><!-- Template:UnsignedIP --> <!--Autosigned by SineBot-->

yeah, just remember that the CONSTANT in the equation, C, isn't just a number, like a million. Instead, it's a "million miles per hour" (for example). So, if you take a pound, or a kilogram, or any unit of mass you want, and mulitply it by a million miles per hour squared, you get a certain amount of energy. I don't have an intuitive conception for why a million miles per hour squared times 1 pound should be an amount of energy though... <span style="font-size: smaller;" class="autosigned">—Preceding [[Wikipedia:Signatures|unsigned]] comment added by [[Special:Contributions/82.120.232.170|82.120.232.170]] ([[User talk:82.120.232.170|talk]]) 17:54, 8 October 2008 (UTC)</span><!-- Template:UnsignedIP --> <!--Autosigned by SineBot-->

:Lets take it in terms of the metric system, for symplicity. Energy is defined, in one instance, as the measure of the change of speed of object of a defined mass over a defined distance. From a unit-based analysis, that means that energy is measured as kilograms times meters squared divided by seconds squared, ( kg*m<sup>2</sup>/s<sup>2</sup> ). That is the inherent definition of energy. The deal is, if you do ANY other combination of measurements, and arrive at the same final units, that is a measure of energy. For example, air pressure*volume is a unit of energy (air pressure can be expressed as force per unit area, and force is mass * acceleration, and volume is displacement cubed, so the if we express pressure as (kg) * (m/s<sup>2</sup>) / (m<sup>2</sup>) * (m<sup>3</sup>) we get, kg*m<sup>2</sup>/s<sup>2</sup>, which is energy. How we calculate energy, as electron-volts or as newton-meters, or as joules, or as liter-atmosphere (these are all energy measurments) is largely dependent on the particlar application. --[[User:Jayron32|Jayron32]].[[User talk:Jayron32|<small>talk</small>]].[[Special:Contributions/Jayron32|<small>contribs</small>]] 18:18, 8 October 2008 (UTC)

:: I've thought of an analogy that might help you: if you could increase your gas mileage by 20% for every 10% under the speed limit that you go (just made that up!!!) you can use that without knowing units for EITHER mileage OR speed limit -- and indeed, our British readers would translate that "tip" into different actual numbers than the Americans...

So if m is kilograms then E would have to be in Joules? [[User:10minnickm|31306D696E6E69636B6D]] ([[User talk:10minnickm|talk]]) 18:29, 8 October 2008 (UTC)
:: No, you're confused by the mistake the poster above made. See my correction of him above. (The poster above is lazy and wants to just cross out units that are the same, for example, when you multiply 55 miles per hour by 2 hours you figure out what units the answer will be by going: [m/h]*[h], ie crossing out the two h's. But you can just as easily ask what distance you go in a minute at that speed).

::The easiest thing to do is play around. Into google put "c=" . Then try "1 lb * c^2". And put "1 kilo * c^2". Then you can ask for the results in different units by saying "in" as I did above ("1 pound * c^2 in horsepower*hours").

::If m is in kg and c is in m/s, the E will be in joules. If you chose different units for c, you'll get different units for E. --[[User:Tango|Tango]] ([[User talk:Tango|talk]]) 21:14, 8 October 2008 (UTC)

:Energy is dimensionally equivalent to Force*Distance, which is equivalent to Mass*Speed*Speed. (See [[Dimensional analysis]] for a better grounding on the subject.) The exact units that Energy comes out in is dependent on the units that you use both for mass and for speed. <math>E=mc^2</math> is always valid, regardless of choice of units for any two of the quantities, however, the units of the third may end up not to be a "conventional" measure (e.g. <math>stone*furlong^2/fortnight^2</math> is a unit of energy, albeit not one that has much use/respect). [[SI]], however, is ''designed'' to be self-consistent, so if you stick to the standard SI units, things tend to work out well, unit-wise. Kilograms, meters, and seconds are all base SI units, so for mass in kg, and speed in m/s, the units of energy work out to the SI unit of energy (joule), because the joule is ''defined'' as equal to <math>1 kg*m^2/s^2</math> (See [[joule#Description]]). -- [[Special:Contributions/128.104.112.147|128.104.112.147]] ([[User talk:128.104.112.147|talk]]) 22:27, 8 October 2008 (UTC)

== safe dough? ==

I live with an Italian, and she's making pizza in four days -- but she's started today. She mixed a spoonful of honey with oil into flour and water and is letting it rise at room temperature, keeping it moist under a moist cloth (which she will keep remoistening over the next four days). This "mother" dough she will use over the course of several pizzas, the first one Saturday, until she's done with it.

My question is: how does this work? I'm not sure what role the "mother" dough has, is it like yeast?

In fact I heard that "sourdough" was created by accident more than a hundred years ago, and since then they have alwyas been reusing the same starter -- they always saved a little to make more. But she's not adding a "starting" piece, she's just starting from honey and oil! So, isn't it just a random toss-up what microbes will start growing? Why is it even safe?

Obviously I'm not asking for any medical or legal advice... <span style="font-size: smaller;" class="autosigned">—Preceding [[Wikipedia:Signatures|unsigned]] comment added by [[Special:Contributions/82.120.232.170|82.120.232.170]] ([[User talk:82.120.232.170|talk]]) 17:14, 8 October 2008 (UTC)</span><!-- Template:UnsignedIP --> <!--Autosigned by SineBot-->

:Actually, the invention of having isolated yeast which you add to the dough to make it rise is a relatively recent invention. For thousands of years, bread was made the same way: You mixed flour, water, and a few seasonings, you sat the mixture on the window sill, and it picked up whatever yeast cultures blew in on the wind, and the bread rose and you baked that. The problem is that some cultures don't taste particularly good. Someone a long time ago figured out that if you find a particularly tasty piece of dough, rather than let future batches of dough get cultured randomly, if you innoculate it with your good-tasting culture, you get equally good tasting bread. This is often accomplished by reserving a "starter" or "mother" culture from each batch of bread, and using it to innoculate the next batch. However, you can also control which cultures take hold in your bread by carefully controling the chemistry of the dough. Cultures are highly sensitive to variations in pH and salinity and other factors, so the specific ratios of say, honey to salt to flour will basically be highly favorable to a single variety of microorganism, and not so much to the others. Once you find a recipe that attracts the right yeast, if you use the same recipe each time, you will end up with the same yeast each time, and thus a fairly consistent bread. --[[User:Jayron32|Jayron32]].[[User talk:Jayron32|<small>talk</small>]].[[Special:Contributions/Jayron32|<small>contribs</small>]] 18:04, 8 October 2008 (UTC)

:(EC) See [[sourdough]], which is exactly what she's making; it should be delicious! The reason it's not a toss-up is that whatever incidental microbes might be hanging around get their asses kicked by the massive amounts of good yeast and bacteria already in the flour. It's only if they run out of food (the honey) that those agents will start to die off and bad ones can take over. Sourdough has been around for thousands of years, and while it is indeed possible to keep a starter going indefinitely, most people just start one from scratch. --[[User:TotoBaggins|Sean]] 18:15, 8 October 2008 (UTC)

:: if I get you right, your response implies 1) flour you buy has lots of good yeast and bacteria in it (and not bad ones), it's made that way? How's that work? 2) if you added honey forever the good ones would always have enough to eat, you could keep it going a month and then bake it... <span style="font-size: smaller;" class="autosigned">—Preceding [[Wikipedia:Signatures|unsigned]] comment added by [[Special:Contributions/82.120.232.170|82.120.232.170]] ([[User talk:82.120.232.170|talk]]) 19:45, 8 October 2008 (UTC)</span><!-- Template:UnsignedIP --> <!--Autosigned by SineBot-->

:::Actually, most of the yeast is present in the air, or on the flour itself. The yeast is just there, it isn't added or anything. It may grow on the grain naturally, or it may just alight on the flour from the air. The determining factor as to which microbe predominates is the local chemistry in the dough. The good yeast, if correctly encouraged to grow, will simply crowd out any nasty critters. --[[User:Jayron32|Jayron32]].[[User talk:Jayron32|<small>talk</small>]].[[Special:Contributions/Jayron32|<small>contribs</small>]] 20:09, 8 October 2008 (UTC)
::::In fact, the use of "ambient" yeast and bacteria in [[lambic beer]] rather than pure yeast cultures is the reason for the rich complex taste of Belgian beer. - <span style="font-family: cursive">[[User:Nunh-huh|Nunh-huh]]</span> 21:03, 8 October 2008 (UTC)
:::1) Correct. It's only natural that microbes that can live on wheat will be found on it in the wild. 2) No, just adding honey won't do it. You also need to add more flour, and water, and keep the bacteria/yeast balance correct. I seem to remember [[Harold McGee]]'s magisterial ''[[On Food and Cooking]]'' covering the details, but I'm sure many baking books do, too. FYI, some cooks have kept starters going for *decades*, though [[refrigeration]] makes that a lot easier. --[[User:TotoBaggins|Sean]] 22:32, 8 October 2008 (UTC)

We cannot rule out that "wild" yeast or fungus might make the dough taste bad or even become toxic. [[User:Edison|Edison]] ([[User talk:Edison|talk]]) 03:53, 9 October 2008 (UTC)
:An archived Ref Desk answer describes some undesirables which might grow in a dough culture : "I suppose we could add a disclaimer about "Ask a baker" or "Ask a mycologist" for bread mold advice. I claim no expertise in this area and only cite info found on the net or from general reading, including a baking textbook, and from baking various types of [[bread]] in the home. The surface of fresh-baked bread is generally free of microorganisms because of the heat of baking, but ropy mold may survive in the interior of bread. The surface can become contaminated while the bread is cooling or being wrapped, while it is sliced, or while it is stored. [http://www.microbiologyprocedure.com/food-microbiology/preservation-of-cereals.htm]. Bread can be attacked by various types of mold. There are over 600 varieties of bread mold. Some of them are listed at [http://gchava.myweb.uga.edu/organisms.html]. Note that the same mold may grow on bread and other substances, and some of these are toxic to humans. In general, some spores are just part of nature. But if mold from bread or other sources has left a high concentration of spores in an area such as your kitchen or your breadbox, it stands to reason that a loaf of bread placed there subsequently might get moldy quicker than if the mold spores were not drifting around. See [[Black bread mold]], [http://www.madsci.org/FAQs/micro/molds.html] at Madsci.org. [[User:Edison|Edison]] 14:59, 14 August 2007 (UTC)" [[User:Edison|Edison]] ([[User talk:Edison|talk]]) 18:49, 9 October 2008 (UTC)

:and, of course, once you've made a few loaves of bread in a kitchen it's pretty thoroughly inoculated with enough yeast on a permanent basis. fungi are remarkably tenacious. I worked in a lab with the fungus [[neurospora crassa]] 20 years ago; i'm still seeing random colonies of neurospora appear around the house where i haven't hit it with the lysol lately, along with the usual household mildew and random gunk. mind you, i didn't grow the fungus at home, that's just from spores which hitched home with me; and moreover, I've moved twice since then. for another example, microbreweries which make beer for on premises consumption but subcontract the bottled product to big breweries (often the case) can specify their recipe, but can't specify their particular strain of yeast, as one particular strain of yeast will colonize a brewery, and you just can't make a batch with a different strain in that premises. (so i've been told at the local microbrewery). see also [[lambic]] for another example of trusting in airborne yeast to produce a palatable and nontoxic product, rather than adding a defined yeast (oh, somebody said that already). [[User:Gzuckier|Gzuckier]] ([[User talk:Gzuckier|talk]]) 06:03, 11 October 2008 (UTC)

== Erronious definition ==

I found this definition for [[Hyperthyroidism]]. I don't believe this is the real definition. I couldn't figure out how to correct it so I am sending it to you.

<small><obvious bit of vandalism removed ... no need to keep that around for posterity --[[Special:Contributions/98.217.8.46|98.217.8.46]] ([[User talk:98.217.8.46|talk]]) 03:13, 9 October 2008 (UTC)></small> <span style="font-size: smallest;" class="autosigned">—Preceding [[Wikipedia:Signatures|unsigned]] comment added by [[Special:Contributions/152.130.7.64|152.130.7.64]] ([[User talk:152.130.7.64|talk]]) 18:27, 8 October 2008 (UTC)</span><!-- Template:UnsignedIP --> <!--Autosigned by SineBot-->

:Yes, that's a bit of vandalism in our hyperthyroidism article. It's fixed now, thanks for pointing it out. - <span style="font-family: cursive">[[User:Nunh-huh|Nunh-huh]]</span> 19:09, 8 October 2008 (UTC)
::Perhaps it's an alternate definition? --[[User:TotoBaggins|Sean]] 22:38, 8 October 2008 (UTC)
:::Perhaps in an alternative universe. - <span style="font-family: cursive">[[User:Nunh-huh|Nunh-huh]]</span> 23:32, 8 October 2008 (UTC)

== Do gas giant even have a flowing water? ==

Do Jupiter, Saturn, Uranus, and Neptune even have a flowing water in it's mantle or it's just a hot dense white-hot vapor if human touch it they will instantly get electricute and roast. All their core is very white-hot, hotter than the surface of the sun.--[[User:Freewayguy|<font color="#00AAAA">SCFR</font>]][[User talk:Freewayguy|<font color="#00AAAA">eew</font>]][[Special:Contributions/Freewayguy|<font color="#00AAAA">ays</font>]] 23:13, 8 October 2008 (UTC)

:To be honest, I'm not sure. Any water in the mantle would be extremely hot, but the pressure may be enough for it to stay liquid. I think it would be all mixed in with everything else rather than flowing, though. A human at those temperatures would be instantly killed, yes, although I don't know why you think they would be electrocuted... --[[User:Tango|Tango]] ([[User talk:Tango|talk]]) 23:20, 8 October 2008 (UTC)

*metallic hydrogen for jupiter and Saturn, certainly they conduct electricity, even Uranus and Neptune the ice to me actually means electronic stuff, hot gluey vapours similar to lightning and molten comets. I don't think is true water the water we use and drink. Those planets is total gas, and landing to me is like going to hell. No solid surface no place to land, from what I've learnt when I was elementary student, if we try to land on it we will just keep sinking trough their interior, until we get to it's nucleus, and even prior to getting to it's center, the strong heat will instantly kill humans, even those dense vapours. Does your saying of liquid mean vapours from the burnt rockets?--[[User:Freewayguy|<font color="#00AAAA">SCFR</font>]][[User talk:Freewayguy|<font color="#00AAAA">eew</font>]][[Special:Contributions/Freewayguy|<font color="#00AAAA">ays</font>]] 23:36, 8 October 2008 (UTC)

::To answer Tango's question, I presume the OP is concerned about that big-ass storm that's been circling Jupiter for at least the last 300 years. And there's plenty of lightning on Jupiter (at least) and presumably the other gas giants. [[User:Matt Deres|Matt Deres]] ([[User talk:Matt Deres|talk]]) 23:42, 8 October 2008 (UTC)

*Even Uranus and Neptune's mantle I beleive is made of deadly "junks" possibly liquid, eelctrical wastes, vapors, perhaps anything.--[[User:Freewayguy|<font color="#00AAAA">SCFR</font>]][[User talk:Freewayguy|<font color="#00AAAA">eew</font>]][[Special:Contributions/Freewayguy|<font color="#00AAAA">ays</font>]] 23:51, 8 October 2008 (UTC)
: I think we can all agree that anyone standing in the core of a gas giant would be killed instantly. The heat and pressure would be extraordinarily deadly. I'm pretty sure you would not live long enough to be electrocuted by a lightning bolt.
: Does that answer your question? [[User:APL|APL]] ([[User talk:APL|talk]]) 01:35, 9 October 2008 (UTC)
::Heat, pressure AND radiation. The gas giants are certainly pretty nasty places. Given that there must be a more or less continuous gradient from the low temperature and pressure of space down to the metallic hydrogen at the center - you'd imagine that there would be some altitude at which liquid water could exist - whether there actually is enough water at that level to form flows is much harder to determine. Probing the conditions deep into that atmosphere is very hard. [[User:SteveBaker|SteveBaker]] ([[User talk:SteveBaker|talk]]) 01:53, 9 October 2008 (UTC)

Are you saying gas giants have a core that is just gas? Not rock? I'd assume enough asteroids fell into them to give them a rocky core, though I hear Saturn is very light. Though I also would assume then that the sun must swallow a lot of asteroids, though why this doesn't make the sun die with iron going into its core, who knows. [[User:Are you ready for IPv6?|Are you ready for IPv6?]] ([[User talk:Are you ready for IPv6?|talk]]) 03:39, 9 October 2008 (UTC)

::::Talking about Earth or gas giant? While Earth have "Greenhouse" or "Icehouse" have nothing to do with gas giants. Earth had icehouse in the past, and once it was cooler when all the continenets is at the South Pole. 100 to 200 million years later, Earth will likely to be greenhouse or hot again. I don't know why you ask this becasue this have nothing to do with the question.--[[User:Freewayguy|<font color="orange">Freeway</font>]][[User talk:Freewayguy|<font color="#FF0000">9</font>]][[Special:Contributions/Freewayguy|<font color="FF0000">1</font>]] 19:58, 9 October 2008 (UTC)

:Gas giants have a solid core, but not because they swallow asteroids, it's because of the immense pressure. [[Special:Contributions/98.221.85.188|98.221.85.188]] ([[User talk:98.221.85.188|talk]]) 04:17, 9 October 2008 (UTC)
:I don't think the core is rock as we know it. In Jupiter's case, at least, it's rather exotic matter ([[metallic hydrogen]] and similar), due to the intense pressure. Regular rock would probably be destroyed long before it reached the core. --[[User:Tango|Tango]] ([[User talk:Tango|talk]]) 10:23, 9 October 2008 (UTC)

::Well, the elements that make up the rock (iron, etc) have to be there somewhere. In these cases, though, it's a matter of scale. Neither Jupiter nor the Sun consume enough asteroids to be relevant when analyzing their overall composition. &mdash; [[User talk:Lomn|Lomn]] 13:01, 9 October 2008 (UTC)
:::Sure, the elements will be there, but not in the forms we're used to. They had plenty of the components of rock from the start, they don't need asteroids for that (well, they make have formed by aggregation of what were essentially asteroids, but that was a long time ago), but those components can't make rock at those temperatures and pressures. --[[User:Tango|Tango]] ([[User talk:Tango|talk]]) 13:36, 9 October 2008 (UTC)
::::I wonder if consumered rock has anything to do with the sun cycles like the sun is supposed to go in cycles where it varies from stronger and weaker to what it puts out. I know that the earth while it did go into cycles of hot to freezing, did used t average a lot hotter but in the last tens of millions of years it gradually really cooled down and then the recent ice age was pretty much its lowest point and then it started warming back up a bit, but still lower than the average. Mars was also supposedly warmer at one time, too. [[User:Are you ready for IPv6?|Are you ready for IPv6?]] ([[User talk:Are you ready for IPv6?|talk]]) 13:46, 9 October 2008 (UTC)
:::::Highly unlikely, again as a matter of scale. There's just not enough foreign material entering the sun to affect things, even if things would be affected in the first place. With that caveat, it's worth noting that even dropping a planet-sized chunk of iron into the sun would likely have no effect. Iron is the fusion stage at which stars quit, yes -- but the presence of iron should have no effect on the [[Proton-proton chain reaction|hydrogen fusion process]] currently ongoing, or the helium fusion that will follow once the hydrogen is exhausted, or the lithium or beryllium or whatever follows that. Iron isn't exothermically fusible, but neither does it poison other fusion reactions. &mdash; [[User talk:Lomn|Lomn]] 14:08, 9 October 2008 (UTC)
::::::[[Metallicity]] does affect how a star behaves, but I agree that on the scale we're talking about the effect would be negligible. I'm not sure much even falls into the sun - to do so it would need to shed almost all its angular momentum, which is pretty difficult to do. --[[User:Tango|Tango]] ([[User talk:Tango|talk]]) 14:18, 9 October 2008 (UTC)

== Time-independent Schrödinger equation: "If <math>\psi</math> is a solution then <math>\psi</math>* is also a solution." ==

In an Introductory Quantum Mechanics book I'm reading, the author often claims that if <math>\psi</math> is a solution then so is its conjugate, and this is stated as being self-evident (leaving me to suspect I'm missing something obvious). Previously, I've only ever encountered complex conjugates when related to an actual complex number, whereas <math>\psi</math> is actually a function <math>\psi(x)</math>, hence my confusion.

The equation is as follows:

<math>
E \psi = -\frac{\hbar^2}{2m}{d^2 \psi \over d x^2} + V(x)\psi
\,</math>

This forms part of a question that asks to show that a time-independent wave function <math>\psi(x)</math> can always be taken to be real, because if a solution isn't real it can just be expressed as a linear combination. Quote: "if <math>\psi(x)</math> satisfies [the above equation], for a given E, so too does its complex conjugate, and hence also the real linear combinations <math>(\psi + \psi*)</math> and <math>i(\psi - \psi*)</math>."

So I basically have two questions:
1) Why can we say <math>\psi*</math> is a solution because <math>\psi</math> is?
2) What else is this question looking for, since it basically gives the answer away when it talks about the linear combinations?

I have the feeling I'm mis-interpreting something here, so I'd be grateful for any clarification or help. [[User:Leucippus89|Leucippus89]] ([[User talk:Leucippus89|talk]]) 23:54, 8 October 2008 (UTC)

: Edit: Just to be clear, this question is about <math>\psi(x)</math> and not <math>\Psi(x,t)</math>. I know that when you solve the Schrödinger equation by separation of variables, the time-dependent function always has complex components which are dependent on t and so could be safely ignored (?) in an equation that's just concerned with x. But that's not what this question's about, and I think that's what's confused me, because I'm not used to seeing complex terms in <math>\psi(x)</math>. [[User:Leucippus89|Leucippus89]] ([[User talk:Leucippus89|talk]]) 00:39, 9 October 2008 (UTC)

:: I suppose <math>E</math>, <math>V</math> and <math>m</math> are all real? If so, it's pretty easy to see that <math>E \psi^* = (E \psi)^*</math> and <math>-\frac{\hbar^2}{2m}{d^2 \psi^* \over d x^2} + V(x) \psi^* = \left( -\frac{\hbar^2}{2m}{d^2 \psi \over d x^2} + V(x) \psi \right)^*</math>. —[[User:Ilmari Karonen|Ilmari Karonen]] <small>([[User talk:Ilmari Karonen|talk]])</small> 02:45, 9 October 2008 (UTC)

= October 9 =

== Sending Sound Wave or Voice Messages to the Past? ==

Is it possible to send sound waves or voice messages to the past? For example, sending sound through time where the sound could be picked up by a local telephone. Can we build a technology to do this? [[Special:Contributions/72.136.111.205|72.136.111.205]] ([[User talk:72.136.111.205|talk]]) 00:19, 9 October 2008 (UTC)

:No. --[[Special:Contributions/98.217.8.46|98.217.8.46]] ([[User talk:98.217.8.46|talk]]) 00:29, 9 October 2008 (UTC)

::Currently, humans have absolutely no ability to send anything through time. It's possible we never will.[[User:CalamusFortis|<font face="Papyrus" color="#000000">Calamus</font>]][[User_talk:CalamusFortis|<font face="Impact" color="#000000">Fortis</font>]] 01:17, 9 October 2008 (UTC)

::: Backwards through time...no. Forwards...yes. Relativity allows 'fast-forwarding' of time. From everything we know about the nature of time, there is absolutely no way to send anything back in time. [[User:SteveBaker|SteveBaker]] ([[User talk:SteveBaker|talk]]) 01:47, 9 October 2008 (UTC)

::::If [[Extrasensory perception]] worked, you might send the voice message by "mental telepathy" to someone in the past who had the right "gift," by some means beyond the knowledge of present science. Sending a sound vibration to an eardrum or making a voice frequency currents flow in phone wires or sending a telegraph message or affecting a computer circuit would require a transfer of energy, which would seem to violate the conservation of mass and energy in the universe both at the present and in the past. Jumping into the realm of science fiction, I suppose a way around this would be if the person 100 years ago left a message that he would send a message to the future, amounting to a certain amount of energy, and you sent a message with an identical amount of energy to the past, and somehow the messages cancelled, conserving mass and energy. That notion would work for letters or for time travellers of equal mass. Of course, any influence from the present to the past would alter the course of events, meaning that things might proceed a little differently in that time space continuum than turned out in ours. [[User:Edison|Edison]] ([[User talk:Edison|talk]]) 03:51, 9 October 2008 (UTC)

:::::The whole idea of time travel is logically inconsistant anyways. If someone could travel in time as freely as in space, it would presuppose some "meta-time" that itself behaved exactly like real time is supposed to behave, but doesn't cuz you are traveling through it. There's a database of causality, and if it isn't time, its something that behaves exactly like it... --[[User:Jayron32|Jayron32]].[[User talk:Jayron32|<small>talk</small>]].[[Special:Contributions/Jayron32|<small>contribs</small>]] 04:13, 9 October 2008 (UTC)

::::::There can be an almost infinite number of parallel universes, with every possible outcome of each quantum choice-point oiccurring, so there is no problem of causality. If I send a message to Kennedy to duck when Oswald shoots, then only that time-space continuum has a second Kennedy term as president, and ours is not affected. If the bread falls jelly side up in one continuum, it falls jelly side down in another. I doubt time travel being possible, because to the best of my knowledge, no chrononaut from the future has ever held a press conference. [[User:Edison|Edison]] ([[User talk:Edison|talk]]) 05:42, 9 October 2008 (UTC)

::::::::Well, never had a ''reliable'' press conference. (There have been claimed chrononauts, e.g. [[John Titor]]). --[[Special:Contributions/98.217.8.46|98.217.8.46]] ([[User talk:98.217.8.46|talk]]) 11:30, 9 October 2008 (UTC)

:(undent) Well, WRT the parallel universe problem, that still presupposes a "branching" causality, which can still only be traced in a forward direction. That forward moving coordinate is still behaving exactly like time. --[[User:Jayron32|Jayron32]].[[User talk:Jayron32|<small>talk</small>]].[[Special:Contributions/Jayron32|<small>contribs</small>]] 18:10, 9 October 2008 (UTC)
::I suppose that in branching causality, a choice point would be whether or not the chrononaut arrived at a certain time and place. [[User:Edison|Edison]] ([[User talk:Edison|talk]]) 18:46, 9 October 2008 (UTC)

:::There is a problem with the branching '[[many worlds]]' hypothesis for time travellers (I've been thinking a lot about this recently because I've been geeking-out on the latest [[Neil Stephenson]] book "[[Anathem]]"). So let's review what is generally said about this:

:::The theory says that in every possible instant in which a quantum event happens ([[Schrodinger's cat]] either dies or doesn't die - to pick a typical example), then the universe splits into two copies that are identical in every way except for that one event. So now there are two universes - one with a live cat in the box and another with a dead one. OK - so now, the "future-you" builds a time machine - goes back in time and yanks the cat out of the box when nobody is looking 10 minutes BEFORE the experiment is started. The idea is that your arrival into the past caused another fork in the universe back before the cat did or did not die - so now there is a third path...that the cat was never in the box in the first place. The future-you who jumped back in time didn't come from the "cat not in the box at all" future - but from...oh - but wait...from BOTH the cat-is-dead and cat-isn't-dead universes? Since those two universes are identical in all other respects - it's pretty reasonable to assume that both of you make identical time machines and jump back to an identical past...at a time when there were VASTLY fewer copies of the universe. Since an insanely large number of quantum events have happened throughout the universe between the cat experiment and the time machine being turned on - there are an insanely large number of time travellers all jumping back to the exact same copy of the universe at (typically) the exact same point in space and time. Only very few potential time travellers would fail to make it back because (through random quantum effects) were too grief-stricken by the pointless death of the kitty - or were just ABOUT to turn on their time machines when a cat leaps onto it and destroys it just seconds before launch.

:::But the fact remains that an almost infinite number of universes have been 'forked' from what was a single universe in the past at the moment in time to which the time machine is aimed. Hence a nearly infinite number of almost identical time travellers would pop into existance at more or less the same instant!

:::This is a big problem for the 'branching' many worlds concept. The reason I mentioned [[Anathem]] is because Stephenson has proposed in it an alternative means for the 'many worlds' thing to happen. I haven't finished thinking my way through the implications of it yet (and all the heavy thinking it requires is slowing my reading of the book to a crawl!) - but essentially he envisages an infinite number of universes proceeding down every possible path through "configuration space" - requiring them to converge as well as diverge! So two universes that have differed only in some very subtle manner could 'collide' and become one universe in which those past differences have quite utterly ceased to matter. This eliminates my complaint rather neatly. But it's not (I think) quite what most many-world-theorists had in mind.

:[[User:SteveBaker|SteveBaker]] ([[User talk:SteveBaker|talk]]) 04:37, 10 October 2008 (UTC)

So...In Stephenson's book (which, we must emphasise, is fiction) - he proposes that we look at the multiverse in "[[configuration space]]". Imagine plotting a graph of (say) the cost of gasoline versus (say) the number of sunspots visibile on the sun...two totally unrelated things. Each point on your graph paper represents one possible combination of price and sunspot count - and if you plot these two things over time - you get a curve that traces around in some bizarre shape - looping back on itself, etc. Well, if you didn't limit yourself to 2D graph paper - you could plot more properties of more things - on 3D graph paper, you could plot price of gas against sunspots against the Dow Jones index. With 4D graph paper, you could add the popularity of the current US president...and if you had a truly INSANE number of dimensions for your hypothetical graph "paper" - you could plot the position, mass, spin, momentum, etc for every single fundamental particle in the entire universe. Since every possible configuration of the universe would be represented by a point somewhere in our bazillion-dimensional graph paper, we can draw a line on that graph that represents the progress of our universe through time...one of Stephensons' characters calls this a "Narrative". We could also plot the narratives of other possible universes through configuration space. OK - so that's Stephenson's idea of configuration space.

When you look at a classical (non-quantum) view of the universe - in which every event is entirely deterministic - the Narrative for our universe would be a L-O-N-G wiggly line snaking it's way through configuration space - looping around a region that's somewhat compact on the bazillion-dimensional graph because so much of the universe is relatively stable. But in a quantum universe like ours where events that are truly random happen and weird superposition phenomena exist - that line branches...just as in the many-worlds interpretation of quantum theory. But viewed in configuration space, it's also possible for two of these Narrative branches to merge back together - if two versions of the universe that differ only by the location of one fundamental particle happen to arrive at the same point in configuration space because that particle ended up in the same place in both of them - making the two universes identical in every respect. This is rather nicer than the conventional "many worlds" picture where universe split into copies of themselves - but never come back together again.

So what we have in Stephenson's rather clever model is that every conceivable point in configuration space (even some crazy, impossible-seeming ones) "exist" in some sense - and each one changes over time by threading a line through that bazillion-dimensional graph - sometimes splitting into two because of quantum randomness - other times merging together because two universes that have (potentially) very different origins and history happen to wind up with identical configurations. (One might argue that this would mess with the memories of living creatures in those universes - but if they have different memories to start with then the two universes can't merge because they are different. Only if every record of past history is somehow erased could two universes join back together. But because configuration space is 100% full of these snaking "narratives" - there are (paradoxically) exactly as many rejoinings as there are splits!

Now - what about time-travel? Well, when a creature who is in a universe that's winding through its narrative line decides to jump "back in time" - they take themselves to a point in configuration space where the universe has a bunch of particles representing the time traveller. That's close (in configuration space) to the narrative that the time traveller's universe passed along - but it's not the same. If the time traveller is super-careful not to kill his grandfather or do anything else to "change the future" (a very tough proposition!) then when he finally leaves the past to return to the present, the universe with the narrative that contains his particles in the past will eventually merge with the narrative of the universe he came from - and he will truly have "done no damage" to the present. If, however (much more likely!) he displaces an air molecule and that has a "butterfly" effect that causes greater and greater divergence of the narrative of this universe from the one he remembers - then perhaps the two narratives will never be able to rejoin.

It could also happen though that he could kill his own grandfather - thereby causing the "new" narrative to diverge still more from our own - but in a million years, the consequences of the time traveller never having lived might be utterly erased by the passage of time such that the original time-line and the new one would become identical in every respect - and rejoin.

But in this peculiar interpretation of the multiverse (which I find exceedingly compelling) - time travel not only might exist - but it definitely does exist - in the sense that somewhere in configuration space, there must be a universe where I suddenly pop into existance for no obvious reason with memories of being a time-traveller. Since all possible configurations "exist" - then for all practical purposes, time travel can happen even if the laws of physics prohibit it. Of course there are also versions of the universe where I pop back into a nearly identical narrative - but my ears have turned to lime jello...or the world of the "past" is entirely populated by giant intelligent pink rabbits.

This is a very weird view of the multiverse - but it's kinda compelling.

[[User:SteveBaker|SteveBaker]] ([[User talk:SteveBaker|talk]]) 12:51, 10 October 2008 (UTC)

:true, an interesting model. and as you say, if all possible universes exist, then there must be an infinity of universes where time travel was invented, and the travelers spilled over into the neighboring universes. but that comes up against the next problem: the chances of us living in a universe where nobody pops up from a parallel where they invented time travel would seem to be close to zero; yet that's where we are. (inspired by the logic that since model universes can be created inside computers or such so much more easily and go through their life cycle so much more quickly that there must be so many more of them, that in all probability we would have to be in one of those, not in the "real" universe.)[[User:Gzuckier|Gzuckier]] ([[User talk:Gzuckier|talk]]) 05:48, 11 October 2008 (UTC)

== Sterling Silver Allergy ==

If I were allergic to sterling silver jewelry, how long would I have to wear a ring around my finger to find out? [[Special:Contributions/98.223.89.7|98.223.89.7]] ([[User talk:98.223.89.7|talk]]) 02:32, 9 October 2008 (UTC)

:I can't comment on sterling silver per se, but I can anecdotally report that my wife is allergic to gold. Its not the impurities in, say, 14K gold, its the gold itself. She's been tested an everything. The higher the Karats of the gold, the worse her reaction is. Even a few second contact with 24 karat gold gives her hives. So it is at least possible to be allergic to a metal like that. As far as how long YOU would take to break out, well, that depends entirely on YOU. Such facts of body chemistry are unique to each individual, and individual reactions cannot be predicted, especially over the internet. If you have questions about possible allergies you may have (and metal allergies are very real), you should see a dermatologist or an allergist and get tested. That is the only way to really know. --[[User:Jayron32|Jayron32]].[[User talk:Jayron32|<small>talk</small>]].[[Special:Contributions/Jayron32|<small>contribs</small>]] 02:43, 9 October 2008 (UTC)

:: This is interesting. How can an inert metal like gold (especially higher purity) trigger an allergic reaction? [[User:Sjschen|Sjschen]] ([[User talk:Sjschen|talk]]) 03:47, 9 October 2008 (UTC)

::: No idea on the mechanics, and if I had never seen it myself, I would not have believed it. We are both chemists by training, and understand the chemistry of gold. Gold is not entirely inert, and there is something there that triggers an allergic reaction in my wife. This is partially a wag, but there may be something about the surface texture of the gold that triggers the reaction; it may not need to actually dissolve into the skin, but the contact triggers a histamine response in my wife. Like I said, it shouldn't work that way, but I can only say that something about gold (and not the alloyants in lower purity gold jewlery, but the gold itself) that triggers the response in my wife. --[[User:Jayron32|Jayron32]].[[User talk:Jayron32|<small>talk</small>]].[[Special:Contributions/Jayron32|<small>contribs</small>]] 04:08, 9 October 2008 (UTC)
::::That's really interesting, especially as you can both take a scientific view on it. Have you tried double-blind experiments? Have you tried the experiment with and without her using whatever soap or lotion she uses on her hands? The latter to examine if the gold somehow acts as a catalyst. There apparently exist gold-based room-temperature catalysts for the oxidation of CO, for instance. [[User:EverGreg|EverGreg]] ([[User talk:EverGreg|talk]]) 08:28, 9 October 2008 (UTC)
:::: I know that Gold can be ionized by a strong oxidizer such as conc. nitric acid, but I doubt sweat and lotions can do anything close to that. Maybe the gold catalyzed rxns in the lotion or soap is indeed the answer. [[User:Sjschen|Sjschen]] ([[User talk:Sjschen|talk]]) 22:28, 9 October 2008 (UTC)
:Just as a totally apocryphal story, I once met a woman with a similar problem, but in her case it was [[Systemic lupus erythematosus|lupus]] and not an allergy (she had a ring tattooed on her finger instead of wearing a wedding band because of a similar problem). My guess is that's more an issue of skin sensitivity and not the metal in itself. One option to test if it's a true allergic symptom is to see if it responds to antihistamines (i.e. [[diphenhydramine]] or somesuch), but it's more likely to be [[Contact dermatitis]] or some other sort of hypersensitivity than a true allergy. [[User:Somedumbyankee|SDY]] ([[User talk:Somedumbyankee|talk]]) 12:41, 9 October 2008 (UTC)
::Yeah, your terminology is probably more correct than mine. I was probably using the term "Allergy" too broadly. The fact is that she has a reaction to gold (she has a platinum wedding band, and that causes no problems for her, and neither does silver or other costume jewlery), and whatever that reaction is is unpleasant for her, as gold causes an itchy rash. The solution for me is I just buy her cheap jewlery. Its a win-win situation for all of us!--[[User:Jayron32|Jayron32]].[[User talk:Jayron32|<small>talk</small>]].[[Special:Contributions/Jayron32|<small>contribs</small>]] 13:10, 9 October 2008 (UTC)

:The original question actually asked how long to wear a ring until they find out. Well pure silver or gold is very weak and usually have alloys in it. Some people are very allergic and can only wear stainless steel, titanium, platinum, palladium, or tungsten, though sometimes I've read forum posts of people who're allergic to those, except I've never heard of a tungsten allergy. Also, there are some online sellers (often in China) who sell what they claim are sterling silver rings and they turn out not to be. [[User:Are you ready for IPv6?|Are you ready for IPv6?]] ([[User talk:Are you ready for IPv6?|talk]]) 13:53, 9 October 2008 (UTC)

::(Not a RefDesk-quality answer...) Jayron, does this happen with other forms of gold, and on other parts of the body? Has she, for instance, "merely" set a gold coin on her arm and got the same reaction? Just curious. Maybe this should be moved to a talk page somewhere. --[[User:DaHorsesMouth|DaHorsesMouth]] ([[User talk:DaHorsesMouth|talk]]) 22:25, 9 October 2008 (UTC)

:Jayron: I really think that if you are both scientists then a double-blind experiment is called for - and I'm very surprised you haven't already tried it. Too many scientists think that science ends at the laboratory door. A while back, my wife was spending a small fortune on bottled water - I maintained that she couldn't tell the difference between the cheap stuff and the expensive stuff - so we did a double-blind experiment. It turned out that not only could she not tell the difference between brands - but she couldn't even tell the difference between the good stuff and tap-water. That experiment saved us a fortune! In your wife's case, I think it's highly likely to be some kind of psychosomatic thing. Gold is just too inert to be a true culprit here. You need two similar sized rings - one gold, one of something else. It should be easy to figure out a double-blind methodology here - have one person take two small boxes labelled "A" and "B" and put one ring into each box - using a coin toss to decide which goes into which box and noting which was which - but not letting the other person know. Then the second person secretly does a coin flip and either swaps over the labels or does not (without looking inside). Then, both of you close your eyes - you pick out the ring from the box marked "A" and rub it onto her skin on her left arm - take the one from "B" and rub it onto her right arm - each time putting the ring back into the same box and closing the lid. Neither of you knows which ring was in which box until the results are in - then you can look into the box to find out - you should really do the experiment several times to be sure the result wasn't a flook. [[User:SteveBaker|SteveBaker]] ([[User talk:SteveBaker|talk]]) 04:10, 10 October 2008 (UTC)
::Yeah, try this conversation on for size "Hey honey, you know that festering rash you get everytime you wear gold? Yeah, that really itchy shit that gets all pusy and nasty? Say, lets do that on purpose". I'll stick to buying her sterling silver jewlery and keep marital harmony going, thanks... --[[User:Jayron32|Jayron32]].[[User talk:Jayron32|<small>talk</small>]].[[Special:Contributions/Jayron32|<small>contribs</small>]] 04:22, 10 October 2008 (UTC)

I may have mentioned this before. I have a friend who has a weird reaction to gold, but only when she drinks alcohol. The skin around her wedding ring goes quite markedly blue. The whole ring finger and that side of her hand is affected. The skin touching her other jewellery or any other material does not do this. To prove the gold is playing some part, she once held a gold chain in the palm of her other hand while she was drinking wine, and her palm also went blue. She removed the chain, and the blue gradually receded, but the ring hand was still blue. It never happens when she's not drinking. I was spooked the first time I saw it. -- [[User:JackofOz|JackofOz]] ([[User talk:JackofOz|talk]]) 04:37, 10 October 2008 (UTC)

:I have seen that before. That is generally due to the formation of copper compounds (which are blue) due to the presense of copper (as a hardener) in most gold jewelery alloys. Apparently, some people have skin chemistry capable of dissolving the copper out of these alloys. My wife has a very different reaction, which is distinctly either allergy, or dermatitis. --[[User:Jayron32|Jayron32]].[[User talk:Jayron32|<small>talk</small>]].[[Special:Contributions/Jayron32|<small>contribs</small>]] 04:45, 10 October 2008 (UTC)
::I know you maintained that it was gold alone that caused your wife's allergy, but is she also allergic to nickel? a two-factor effect with gold + nickel could be one hypothesis. [[User:EverGreg|EverGreg]] ([[User talk:EverGreg|talk]]) 21:11, 10 October 2008 (UTC)

== Perpetuating genes ==

Are there any known ways I can perpetuate my genetic stock without myself reproducing? Any that won't contribute to overpopulation? Also, are there any known ways of determining whether my genetic stock is above-average and worth making an effort to perpetuate? I have Asperger Syndrome, if that makes any difference. [[User:NeonMerlin|<span style="background:#000;color:red;border:#0f0 solid;border-width:1px 0">Neon</span>]][[User talk:NeonMerlin|<span style="background:#0f0;color:#000;border:red solid;border-width:1px 0">Merlin</span>]] 03:20, 9 October 2008 (UTC)

:You could store your own semen. There are many sperm banks and other services that will do this for you. If you are female, you could also have eggs extracted and stored, but that involves invasive surgery, and is far riskier and less pleasant than the male method for extraction of reproductive genetic information. --[[User:Jayron32|Jayron32]].[[User talk:Jayron32|<small>talk</small>]].[[Special:Contributions/Jayron32|<small>contribs</small>]] 03:22, 9 October 2008 (UTC)

::Note that the technology for long-term storage of unfertilized eggs is very new and has resulted in very few live births (I think I read about a first within the last year, but most IVF clinics don't do it yet). The exact reason why is not all that clearly understood, but freezing and the fertilization process don't work all that well together. Fertilized eggs can be stored easily, as can semen. [[User:Somedumbyankee|SDY]] ([[User talk:Somedumbyankee|talk]]) 08:47, 9 October 2008 (UTC)

:You're asking some pretty tough questions, and it's hard to give suggestions here without bias. I suggest looking at [[genetic diversity]], [[natural selection]], and [[genetic testing]] to start you on the path to finding your own answers. [[User:Sjschen|Sjschen]] ([[User talk:Sjschen|talk]]) 04:01, 9 October 2008 (UTC)

:An alternative is to support one's extended family as they have much the same genes as yourself. This is a tried and tested way of propagating genes in the animal world, with examples ranging from insects to mammals. [[User:EverGreg|EverGreg]] ([[User talk:EverGreg|talk]]) 08:34, 9 October 2008 (UTC)

::This was what I was going to recommend. If your sister reproduces (if you have a sister), it's pretty similar genetic stock as your own, so at times it might be worth (from an evolutionary point of view) working to enhance her reproductive fitness even at the detriment of your own. See [[kin selection]]. --[[Special:Contributions/98.217.8.46|98.217.8.46]] ([[User talk:98.217.8.46|talk]]) 11:20, 9 October 2008 (UTC)

::As for "above average"... what do you really mean by that? You'd need to define your system of genetic worth first. It's not intuitive, and frankly we would probably disagree on what traits were most "desirable" and even some that were less "desirable". In the long run though your individual concerns about the worth of your genetics will play little role in the overall gene pool unless your reproduction rate is significantly differential from the average (either you produce more or they produce less). See [[eugenics]]. --[[Special:Contributions/98.217.8.46|98.217.8.46]] ([[User talk:98.217.8.46|talk]]) 11:20, 9 October 2008 (UTC)

:Although encouraging your relatives to reproduce will perpetuate your stock without your reproducing, it will still contribute to overpopulation. Theoretically, you could try to prevent other people from reproducing, but that would have a negligible effect on the portion of people who are related to you. Your only real solution is to go ahead and reproduce, and counter the effect on overpopulation by preventing others from having kids, or by increasing the capacity of the world. Because people in poverty reproduce more, I suggest donating to a charity that helps stop poverty, such as [[microcredit]]. If you donate a significant amount of money, it will be overkill, but I recommend that you do so anyway. — [[User:DanielLC|DanielLC]] 14:43, 9 October 2008 (UTC)

:"Goodness" for a gene is really determined only by it's ability to spread and continue. Genes which don't make it into the next generation for some reason are "bad". So there is no way to indirectly measure goodness or badness of someone's genes. If they succeed in reproducing (preferably as many times as possible - and continuing for as many generations as possible) - then they are functioning well. A decision to not have children (no matter how socially good that is) is an indication that your genes have already failed in their goal to reproduce. Sooner or later, someone will come along with a higher sex drive or a higher maternal/paternal instinct that manages to overpower any hint of a desire to keep the earth's population in check. When that happens, their genes will spread faster than yours and your genes have failed and were therefore not as "good" as the ones that wiped them off the face of the gene pool. However, humans are (IMHO) gradually seeing more influence from memetic evolution than genetics. [[Meme]]s are also able to propagate, evolve and reproduce - and in humans, a meme can have a greater role in driving us than our genes do. So perhaps the 'meme' that's telling us to save the planet is more powerful than the gene that's telling us to fill the planet from edge to edge with human beings. It is arguable, therefore, that you are doing a better job of passing on your best memes to the next generation than you are at passing on genes. So - become a teacher, or a writer or a musician - or merely answer questions on the ref desk in a way that reinforces the ideas (memes) that are worth passing on. [[User:SteveBaker|SteveBaker]] ([[User talk:SteveBaker|talk]]) 03:53, 10 October 2008 (UTC)

:You might find [[Nature versus nurture]] an interesting read as well. [[User:Somedumbyankee|SDY]] ([[User talk:Somedumbyankee|talk]]) 15:05, 10 October 2008 (UTC)

:in the Big Picture, you in all probability do not carry any genes which are not represented in the entire human gene pool at least several million times. and furthermore, if you did carry any rare mutations, the vast majority of them would be deleterious. so you can rest assured that even if you die without issue, your genetic components will not be lost to humanity. [[User:Gzuckier|Gzuckier]] ([[User talk:Gzuckier|talk]]) 05:41, 11 October 2008 (UTC)

== Is our planet's core mostly iron because of boyancy? ==

?? [[Special:Contributions/98.221.85.188|98.221.85.188]] ([[User talk:98.221.85.188|talk]]) 04:18, 9 October 2008 (UTC)
:[[Inner core]] is a pretty good read. One of the main driving forces behind the assumption that the core is mostly iron is that we know what everything else in the solar system is made of, and we know what our planet should be made of, and there's not nearly enough iron anywhere else on the planet to match the rest of the solar system. Ergo, the iron has to be somewhere, so its gotta be in the core. At least, that's how our article explains it. Its a fairly logical analysis of the situation. --[[User:Jayron32|Jayron32]].[[User talk:Jayron32|<small>talk</small>]].[[Special:Contributions/Jayron32|<small>contribs</small>]] 04:33, 9 October 2008 (UTC)
::However, from the article [[Earth's magnetic field]]: Citing oceanic basalt 3He/4He ratios [5] and other evidence, J. Marvin Herndon et al contend that the inner core of the Earth is not iron but much denser atoms. --[[User:Ayacop|Ayacop]] ([[User talk:Ayacop|talk]]) 18:27, 9 October 2008 (UTC)
:::That's not what our articles say though. Our articles claim that the core is mostly Iron. And besides, what would these denser atoms be? [[Special:Contributions/98.221.85.188|98.221.85.188]] ([[User talk:98.221.85.188|talk]]) 14:37, 10 October 2008 (UTC)

== Balancing equations ==

I can't for the life of me balance this equation:
H<sub>2</sub>S + SO<sub>2</sub> ---> S<sub>8</sub> + H<sub>2</sub>O . The S<sub>8</sub> in the product's side is throwing me off! I thought I knew how to do this, until I attempted this problem. Can someone explain to me how to do this? I've been at it for far too long! <span style="font-size: smallest;" class="autosigned">—Preceding [[Wikipedia:Signatures|unsigned]] comment added by [[Special:Contributions/134.241.222.116|134.241.222.116]] ([[User talk:134.241.222.116|talk]]) 05:16, 9 October 2008 (UTC)</span><!-- Template:UnsignedIP --> <!--Autosigned by SineBot-->
:Note that that there must be twice as much H as O on the right. The S is just along for the ride. The coefficients are not low. I guarantee the equation will balance. [[User:Edison|Edison]] ([[User talk:Edison|talk]]) 05:30, 9 October 2008 (UTC)

::In other words, if the left hand side of the balanced equation is ''A''H<sub>2</sub>S + ''B''SO<sub>2</sub> where ''A'' and ''B'' are integers, then you must have ''A''=2''B'' in order to get the hydrogen and oxygen to balance, and also ''A''+''B'' must be a multiple of 8 so that you have a whole number of S<sub>8</sub> molecules on the right hand side. [[User:Gandalf61|Gandalf61]] ([[User talk:Gandalf61|talk]]) 09:06, 9 October 2008 (UTC)

I still can't get it! Can you just tell me what it is? <span style="font-size: smallest;" class="autosigned">—Preceding [[Wikipedia:Signatures|unsigned]] comment added by [[Special:Contributions/69.16.88.147|69.16.88.147]] ([[User talk:69.16.88.147|talk]]) 11:15, 9 October 2008 (UTC)</span><!-- Template:UnsignedIP --> <!--Autosigned by SineBot-->

:Also note that, since ''A''=2''B'', ''A''+''B''=2''B''+''B''=3''B''. Since 8 is not divisible by 3, the fact that 3''B'' must be a multiple of 8 implies that ''B'' itself must be a multiple of 8. In particular, the smallest value that could possibly be a solution is ''B''=8. Why not try plugging it in and see if it works out? —[[User:Ilmari Karonen|Ilmari Karonen]] <small>([[User talk:Ilmari Karonen|talk]])</small> 11:45, 9 October 2008 (UTC)

::To restate the above, since the water/SO<sub>2</sub> ratio requires there to be 3 sulfurs on the left side of the equation, and the S<sub>8</sub> is on the right, the answer must involve a number of sulfurs that is divisible BOTH by 3 and by 8. The lowest number that works for that is 24, so try to work out a solution that involves 24 sulfurs. That should put you in the right direction. An alternative method would involve solving this via the half-cell method. The reaction is a [[Synproportionation]] reaction, where sulfur in 2 oxidation states react to form a third oxidation state of sulfur. Track the oxidation numbers and see for yourself. You could try setting up two half-cell equations and balancing each and recombing them. This method is described [http://scienceworld.wolfram.com/chemistry/HalfReactionEquationBalancing.html here] and an example is put forward [http://www.mpcfaculty.net/mark_bishop/Redox_balancing.htm here]. If the brute force method using 24 sulfurs doesn't work, try the half-cell method.--[[User:Jayron32|Jayron32]].[[User talk:Jayron32|<small>talk</small>]].[[Special:Contributions/Jayron32|<small>contribs</small>]] 13:03, 9 October 2008 (UTC)

:My recommendation to my students - ignore the requirement for integers until the end. After balancing the oxygen and hydrogen, you have 3 sulfur atoms on the left, with S<sub>8</sub> on the right. So you require 3/8ths of S<sub>8</sub>. At the end, you can't have fractions, so then multiply <s>everything</s> all the coefficients by 8. I find this method a lot more intuitive than some of the above. --[[User:Bennybp|Bennybp]] ([[User talk:Bennybp|talk]]) 14:56, 9 October 2008 (UTC)

== Ovulation ==

Why does the human body temperature increase whilst ovulating?[[Special:Contributions/90.210.162.166|90.210.162.166]] ([[User talk:90.210.162.166|talk]]) 08:20, 9 October 2008 (UTC)
:You may want to read Wikipedia's article on [[Ovulation]]. Having read it myself, I could posit an educated guess that ovulation is controlled by [[Pituitary gland|pituitary hormones]], as mentioned in that article. The pituitary gland also controls [[metabolism]] and [[homeostasis]] in the body, and one of its main jobs is the regulation of your body temperature via metabolic control. My guess is that changes in body temperature are an indication that the pituitary is firing up in some way, which is a likely indication that it has released whatever the pre-ovulatory hormones are. This is largely a guess based on some intuitive readings of the articles on Ovulation and the Pituitary gland, but it makes sense to me. Does anyone else have a more concrete answer? --[[User:Jayron32|Jayron32]].[[User talk:Jayron32|<small>talk</small>]].[[Special:Contributions/Jayron32|<small>contribs</small>]] 12:52, 9 October 2008 (UTC)
: From "Review of Medical Physiology" (William Ganong), the temperature rise at ovulation is probably due to the increase in [[progesterone]] secretion. Progesterone is thermogenic (i.e. increases metabolic rate and generates more heat). [[User:Axl|<font color="#808000">'''Axl'''</font>]] <font color="#3CB371">¤</font> <small>[[User talk:Axl|<font color="#6B8E23">[Talk]</font>]]</small> 18:03, 9 October 2008 (UTC)

==True or False? Attack of the mutant 6 foot man eating catfist?==

There is this story about a scientist who found a man eating catfish. As reported in the Newspaper.

http://www.thesun.co.uk/sol/homepage/news/weird/article1784470.ece

Is this story scientifically true? [[User:Ohanian|Ohanian]] ([[User talk:Ohanian|talk]]) 10:50, 9 October 2008 (UTC)

:Well it certainly doesn't seem scientifically supported. All evidence given seems to be from what local natives and one reporter say—which generally means it is anecdotal evidence, the stuff of legend (no matter whether your locals are in India or Missouri). Personally I would want to see why the reporter fellow ruled out animals better known to have attacked humans in the past (crocodiles, snakes, etc.) before jumping to a new theory. --[[Special:Contributions/98.217.8.46|98.217.8.46]] ([[User talk:98.217.8.46|talk]]) 11:10, 9 October 2008 (UTC)

:Ha ha I read that as a scientist finding a man who was eating a catfish, rather than a man-eating catfish. I'd go with 98.217's reasoning, though a man eating a catfish would be pretty plausible - if not exactly newsworthy. [[Special:Contributions/194.221.133.226|194.221.133.226]] ([[User talk:194.221.133.226|talk]]) 12:29, 9 October 2008 (UTC)

: [[The Sun]] isn't really world renowned for its fact-checking. [[User:APL|APL]] ([[User talk:APL|talk]]) 12:48, 9 October 2008 (UTC)
:: If you see the cover of The Sun at supermarkets, it's usually hypin some biblical prophecy that it says is about to happen. [[User:Are you ready for IPv6?|Are you ready for IPv6?]] ([[User talk:Are you ready for IPv6?|talk]]) 13:58, 9 October 2008 (UTC)
::: You should have put an "NSFW" warning on that. ;-) [[User:Axl|<font color="#808000">'''Axl'''</font>]] <font color="#3CB371">¤</font> <small>[[User talk:Axl|<font color="#6B8E23">[Talk]</font>]]</small> 18:07, 9 October 2008 (UTC)
:::I have seen catfish which weighed 50 pounds, and reliable witnesses have told of seeing larger ones living at the bottom of U.S. rivers just downstream from hydroelectric dams, eating the chopped fish who went through the turbines. As bottom feeders, catfish would certainly eat human bodies which had been placed in a river. All in all, it seems plausible but certainly not proved by a photo in a tabloid. I would have no trouble believing a report of one half the weight they claim. I could place a minnow that large in someone's hands in a Photoshopped picture. Seeing is not believing.
::::I don't know that much about catfish in general, but I can't quite imagine one eating a live human. I know that they would eat dead meat, so a dead human body wouldn't be that unlikely, but the live human...nonetheless, we could have a story about three 6 foot men eating a catfish, judging by that picture :-) [[User:Nyttend|Nyttend]] ([[User talk:Nyttend|talk]]) 22:01, 9 October 2008 (UTC)
:6-foot and 161 lbs may be a record for that species of catfish but some species get much larger. The record [[Mekong giant catfish]] was "9 feet in length and weighing 293 kg (646 lb)." [[User:Rmhermen|Rmhermen]] ([[User talk:Rmhermen|talk]]) 23:42, 9 October 2008 (UTC)

Snopes has a [http://www.snopes.com/photos/animals/catfish.asp couple] of [http://www.snopes.com/critters/lurkers/catfish.asp stories] about huge catfish that may be of interest. [[User:Matt Deres|Matt Deres]] ([[User talk:Matt Deres|talk]]) 00:28, 10 October 2008 (UTC)

== Quantum: why for entangled state but not separable states ==

I think there should be no difference between the entangled states and [[separable states]] (entangled states are just quantum states with special "pattern" and nothing else), so I speculate: if we measure one of the two entangled particle, the other will [[wave function collapse|collapse]] instantly (just like this [[EPR_paradox#Measurements_on_an_entangled_state|experiment]]), then how about measuring one of the two separated (non-entangled) particle? If we measure one of the two particle in separable state, does the other separated particle also collapse instantly? do whole of the other separated particles in our unverse collapse together with it (as the distance doesn't matter) <s>simutaniously</s> simultaneously and instantly? Can I say that all the other separated particles in our universe will actually collapse but just our sensor/device can't distinguish <s>wheather</s> whether they have collasped or not since they are in separable state? - [[User:Justin545|Justin545]] ([[User talk:Justin545|talk]]) 12:07, 9 October 2008 (UTC)

=== Wave Function Collapse of Entangled Separable State ===
:Suppose we have two particles <math>A</math> and <math>B</math> their respective states are
::<math>|\psi\rangle_A=\sqrt{0.2}|0\rangle_A+\sqrt{0.8}|1\rangle_A</math>
::<math>|\psi\rangle_B=\sqrt{0.3}|0\rangle_B+\sqrt{0.7}|1\rangle_B</math>
:The state of the composite system of <math>A</math> and <math>B</math> is
::<math>|\psi\rangle_A\otimes|\psi\rangle_B=(\sqrt{0.2}|0\rangle_A+\sqrt{0.8}|1\rangle_A)\otimes(\sqrt{0.3}|0\rangle_B+\sqrt{0.7}|1\rangle_B)</math>
::<math>|\psi\rangle_A|\psi\rangle_B=\sqrt{0.06}|0\rangle_A|0\rangle_B+\sqrt{0.14}|0\rangle_A|1\rangle_B+\sqrt{0.24}|1\rangle_A|0\rangle_B+\sqrt{0.56}|1\rangle_A|1\rangle_B</math>
:If we try to measure the state of particle <math>A</math> of <math>|\psi\rangle_A|\psi\rangle_B</math> and get state <math>|1\rangle_A</math>, it means <math>|\psi\rangle_A|\psi\rangle_B</math> collapses to either <math>|1\rangle_A|0\rangle_B</math> or <math>|1\rangle_A|1\rangle_B</math>. Besides, the probability of finding particle <math>B</math> in state <math>|1\rangle_B</math> is
::<math>P\left(|1\rangle_B\Big||1\rangle_A\right)=\frac{P(|1\rangle_A\cap|1\rangle_B)}{P(|1\rangle_A)}</math> (according to [[conditional probability]] <math>P(B \mid A) = \frac{P(A \cap B)}{P(A)}</math>)
:where
::<math>P(|1\rangle_A\cap|1\rangle_B)=P(|1\rangle_A|1\rangle_B)=|\sqrt{0.56}|^2=0.56</math>
::<math>P(|1\rangle_A)=|\sqrt{0.24}|^2+|\sqrt{0.56}|^2=0.8</math>
:therefore,
::<math>P\left(|1\rangle_B\Big||1\rangle_A\right)=\frac{0.56}{0.8}=0.7=|\sqrt{0.14}|^2+|\sqrt{0.56}|^2=P(|1\rangle_B)</math>
:Similarly, if we try to measure the state of particle <math>A</math> of <math>|\psi\rangle_A|\psi\rangle_B</math> and get state <math>|0\rangle_A</math>, it means <math>|\psi\rangle_A|\psi\rangle_B</math> collapses to either <math>|0\rangle_A|0\rangle_B</math> or <math>|0\rangle_A|1\rangle_B</math>. Besides, the probability of finding particle <math>B</math> in state <math>|1\rangle_B</math> is
::<math>P\left(|1\rangle_B\Big||0\rangle_A\right)=\frac{0.14}{0.06+0.14}=0.7=P(|1\rangle_B)</math>
:The above illustration shows that we are not able to distinguish whether the state of particle <math>B</math> has collapsed or not, because no matter the state of particle <math>A</math> we measured is <math>|0\rangle_A</math> or <math>|1\rangle_A</math>, the state of particle <math>B</math> always collapses to <math>|1\rangle_B</math> with probability <math>0.7</math><!-- even if the wave function <math>|\psi\rangle_A|\psi\rangle_B</math> really collapsed -->. Therefore, we can say the particle <math>B</math> DOSE collapse instantly (and so do all other particles in the universe) when we measure particle <math>A</math>, but we just have no way to emphasize that.
<!-- :0.24/0.8 + 0.56/0.8 = 0.3 + 0.7 = 1 -->
<!-- :0.06/0.2 + 0.14/0.2 = 0.3 + 0.7 = 1 -->
:- [[User:Justin545|Justin545]] ([[User talk:Justin545|talk]]) 06:06, 10 October 2008 (UTC)

== what is the best way to structure a case study presentation on a medical patient ? ==

I have to present a case study to a large, mixed group of medical and allied-medical professionals. I have chosen my subject (the patient) and would like opinions on how best to structure and present the information and how to encourage discussion.[[User:Mollyisthedog1|Mollyisthedog1]] ([[User talk:Mollyisthedog1|talk]]) 13:21, 9 October 2008 (UTC)

: Are you planning to use Powerpoint? See "[[Medical history]]". Start with age & sex. [[Presenting complaint]]. [[History of presenting complaint]]. Past medical history. Drugs & allergies. Occupation. Social circumstances. Smoking & alcohol. [[Clinical examination]] findings. Relevant investigations. Treatment. Information about the specific disease. In smaller groups, I often invite individuals to comment on differential diagnosis and proposed management during the presentation. [[User:Axl|<font color="#808000">'''Axl'''</font>]] <font color="#3CB371">¤</font> <small>[[User talk:Axl|<font color="#6B8E23">[Talk]</font>]]</small> 18:16, 9 October 2008 (UTC)

::Have you seen [[House (TV series)]]?

: Just a thought, it may be worthwhile to present a brief abstract and overview before you plunge into details. Much of your audience will likely be used to reading journal articles, so following that structure (summary->methods->data->interpretation) will make the talk easier to follow for those familiar with the format. If the case study involves several cycles of "let's try this test, no that didn't work" it may make sense to have an overall summary, then do methods/data/discussion for each angle considered. [[User:Somedumbyankee|SDY]] ([[User talk:Somedumbyankee|talk]]) 15:03, 10 October 2008 (UTC)

== solar power roof installations and having to do roof maintenance ==

Hi,

When you install a solar panel on your roof, and then have to replace the roof shingles for routine maintenance, how do the solar panels get in the way? Do you have to take the solar panels off, replace the shingles, and then reinstall the solar panels? What's the deal?

I feel irritated that all the solar roof websites I've read don't discuss this issue.

Thanks

--[[User:InverseSubstance|InverseSubstance]] ([[User talk:InverseSubstance|talk]]) 18:31, 9 October 2008 (UTC)

:Hm. I don't know much about this, but perhaps calling one of the companies directly may get you a live person, who could answer any specific questions that you have? Its a start... --[[User:Jayron32|Jayron32]].[[User talk:Jayron32|<small>talk</small>]].[[Special:Contributions/Jayron32|<small>contribs</small>]] 19:20, 9 October 2008 (UTC)

:[[Asphalt shingle]]s are now available with rated lifetimes of forty years or more; steel roofing is often good for a hundred years. A properly designed and built roof that doesn't use low-quality shingles will almost certainly outlast the photovoltaic system on top. Proper installation of the solar panels may even slightly increase the lifespan of the roof by providing a small amount of extra protection from the elements. What type of 'routine maintenance' are you expecting to have to do that would require removal and replacement of shingles?
:Interestingly, one can also purchase '[[solar shingles]]'&mdash;photovoltaic panels which replace shingles altogether. [[User:TenOfAllTrades|TenOfAllTrades]]([[User_talk:TenOfAllTrades|talk]]) 19:33, 9 October 2008 (UTC)
::The questioner could be asking about a solar water heater, and the removal of shingles might not be for maintenance of the shingles but maintenance of the roof structure. It's going to be hard to answer the question without knowing a lot more about the structure of the roof, how large the panel is, how and where it is located. It's good practice always to get at least three quotations for any kind of building work and the companies should be asked whether they have encountered a similar situation before and how they dealt with it. [[User:Itsmejudith|Itsmejudith]] ([[User talk:Itsmejudith|talk]]) 13:35, 10 October 2008 (UTC)

== Watch question ==

When was the [http://alanwatch.homestead.com/page8point5.html jump hour] function invented for clocks and watches? I imagine it must be quite old ([[:Image:Jumphour.jpg|here's]] a watch from the 1890s with the ability), but was curious. --[[Special:Contributions/140.247.42.160|140.247.42.160]] ([[User talk:140.247.42.160|talk]]) 20:10, 9 October 2008 (UTC)

== Unit of Measures ==

Would UOM KOhm be the same as kOhm <small><span class="autosigned">—Preceding [[Wikipedia:Signatures|unsigned]] comment added by [[User:A01534|A01534]] ([[User talk:A01534|talk]] • [[Special:Contributions/A01534|contribs]]) 20:27, 9 October 2008 (UTC)</span></small><!-- Template:Unsigned --> <!--Autosigned by SineBot-->

:I would think so. A lowercase k is the correct abbreviation for the [[SI prefix]], "kilo-", but "K" is fairly common. I don't know anything else it could mean (in computing it sometimes means 1024 times, but that's non-standard). --[[User:Tango|Tango]] ([[User talk:Tango|talk]]) 20:54, 9 October 2008 (UTC)

::I always found that slightly confusing. I had thought capitals were for "bigger" [[SI prefix| prefixes]], and lower-case was for "smaller" prefixes. That seems to be the pattern most of the time - mega vs. milli, peta vs. pico, etc. But kilo, deca, and hecto don't. I guess it would be asking too much to be completely logical.... But yes, K probably means k in your question - there is no (SI) prefix for a capital K. --[[User:Bennybp|Bennybp]] ([[User talk:Bennybp|talk]]) 22:07, 9 October 2008 (UTC)

:::What you have to remember is that all the prefixes from milli- to kilo- are part of the original metric system while the ones from mega- upwards (and from micro- downwards) are later additions that have come in several batches over time. Presumably the forms "kg" and "km" were well established by the time that M- for mega- was introduced, and it wasn't until there were several more prefixes that the pattern of "higher prefixes get capitalized" was really established. So it makes sense that there was no pressure to change kilo- to be represented with a capital K. --Anonymous, 01:08 UTC, dekaOctober :-) 2008.

::::Along those same lines, the M- for mega- is also obviously influenced by the m- prefix already being in use. The next step there led to the use of μ- for micro-. Had more size prefixes started with k's, you'd likely see k-, K-, and κ- prefixes instead. Personally, I think using the Greek symbol was a dumb idea, but they no doubt didn't think of the hassle it was cause folks trying to use it on standard keyboards (and typewriters!). I would have gone with i- (for itsy-). ;-) [[User:Matt Deres|Matt Deres]] ([[User talk:Matt Deres|talk]]) 10:48, 10 October 2008 (UTC)

:::::They probably rationalized it by assuming the letter 'u' could be used in place of the 'μ' without loss of clarity in typewritten documents. (No SI prefix uses upper- or lowercase 'u'.) Heck, I still use that shortcut&mdash;I've got Word set to autocorrect any instance of um to μm for me as I type.

:::::I'd still say it's a better solution than the one adopted by some (mostly electrical) engineers. They use ''mm'' as a prefix (millimilli-) in lieu of mu, which is just asking for trouble. (Did you mean 1 mF or 1 mmF? Or do you want me to use a capacitor 1 mm long?) [[User:TenOfAllTrades|TenOfAllTrades]]([[User_talk:TenOfAllTrades|talk]]) 13:50, 10 October 2008 (UTC)

:::::In fact the use of "u" in place of µ was officially approved by the ISO in 1974, but later withdrawn as character sets including µ became more commonly available. --Anonymous, 18:57 UTC, October 10, 2008.

::::::See, all this confusion could have been avoided if they had just asked me in the first place! That way, at least I would have understood. (Of course I was only born in 1985, but that's what time machines are for!) :) --[[User:Bennybp|Bennybp]] ([[User talk:Bennybp|talk]]) 19:47, 10 October 2008 (UTC)
:::::::Then you'll appreciate what King [[Alfonso X of Castile]] (known as "Alfonso the Wise") said: ''Had I been present at the creation, I would have given some useful hints for the better ordering of the universe''. -- [[User:JackofOz|JackofOz]] ([[User talk:JackofOz|talk]]) 20:28, 10 October 2008 (UTC)
: "Ohm" as a unit of measure is not capitalized: hence "kohm". [[User:Axl|<font color="#808000">'''Axl'''</font>]] <font color="#3CB371">¤</font> <small>[[User talk:Axl|<font color="#6B8E23">[Talk]</font>]]</small> 18:57, 10 October 2008 (UTC)

::Of course, if one were being pedantic, kohm is no more acceptable than kgram, kmeter, or kamp. It's either k&Omega; or kilohm. [[User:TenOfAllTrades|TenOfAllTrades]]([[User_talk:TenOfAllTrades|talk]]) 19:05, 10 October 2008 (UTC)
:::It would seem your spelling is correct, but what happened to the other 'o'? --[[User:Tango|Tango]] ([[User talk:Tango|talk]]) 19:10, 10 October 2008 (UTC)

::::It seems to be an arbitrary decision by the powers-that-be (I presume the [[BIPM]]). For ease of use, the doubled vowel was removed from kilohm and megohm (instead of using kiloohm and megaohm). A similar change was made with the [[are]], giving us areas in hectares rather than hectaares. Not all double vowels are forbidden, however&mdash;kiloamperes and megaampere are cool. I imagine that there's a formal document spelling all this out somewhere, but I can't bring myself to hunt it down at the moment. [[User:TenOfAllTrades|TenOfAllTrades]]([[User_talk:TenOfAllTrades|talk]]) 20:14, 10 October 2008 (UTC)

:::::Googling bipm.org reveals 3 instances of "[http://www.google.co.uk/search?client=firefox-a&rls=org.mozilla%3Aen-GB%3Aofficial&channel=s&hl=en&q=kiloohm+site%3Abipm.org&meta=&btnG=Google+Search kiloohm]" and none of "[http://www.google.co.uk/search?hl=en&safe=off&client=firefox-a&channel=s&rls=org.mozilla%3Aen-GB%3Aofficial&hs=cBH&q=kilohm+site%3Abipm.org&btnG=Search&meta= kilohm]". The [http://www.bipm.org/en/si/si_brochure/ SI brochure] itself says nothing about removing vowels. It seems to me that the shortened forms, although widespread, are not sanctioned by the [[CGPM]], but some national bodies like the [[NIST]] choose to define their own variants. --[[User:Heron|Heron]] ([[User talk:Heron|talk]]) 10:40, 11 October 2008 (UTC)

== trying to remember the name of a certain stone ==

hi I have been trying to remember the name of a certain stone that we used to see alot of when we were kids. its a flacky stone that is black and foung in abundance in caves. i believed it was called "mika" but a search of it came up empty. the best way to describe it is that it usually comes in small or large chunks but can easily be split into little flakes and crumbles verry easily. If you could please get back to me it would be verry much apreciated. My email is [Redacted] <span style="font-size: smaller;" class="autosigned">—Preceding [[Wikipedia:Signatures|unsigned]] comment added by [[Special:Contributions/99.241.82.198|99.241.82.198]] ([[User talk:99.241.82.198|talk]]) 20:51, 9 October 2008 (UTC)</span><!-- Template:UnsignedIP --> <!--Autosigned by SineBot-->

:(I've removed your email address to protect you from spam - people will reply here.) I don't know much about rocks, so you'll have to wait for someone else to come along, but you can look through [[List of rock types]] and see if anything jumps out at you, if you like. --[[User:Tango|Tango]] ([[User talk:Tango|talk]]) 20:57, 9 October 2008 (UTC)

:[[Mica]]? [[User:Saintrain|Saintrain]] ([[User talk:Saintrain|talk]]) 20:59, 9 October 2008 (UTC)
::And if the variety you were seeing was black, [[Biotite]]. [[User:Deor|Deor]] ([[User talk:Deor|talk]]) 02:58, 10 October 2008 (UTC)

:Another possibility is [[flint]] - but I was also thinking mica when I first read the question. Flint can be flaked, but mica is more "crumbly". [[User:Franamax|Franamax]] ([[User talk:Franamax|talk]]) 06:38, 10 October 2008 (UTC)

== Venus axial tilt ==

Which one is right? Is Venus upside-down tilt axis of 177 degs, or normal upright of 2.6 degs. Old books say 2.6 degs axial tilt, but ewer study say Venus is upside down of 177 degs.--[[User:Freewayguy|<font color="orange">Freeway</font>]][[User talk:Freewayguy|<font color="#FF0000">9</font>]][[Special:Contributions/Freewayguy|<font color="FF0000">1</font>]] 22:28, 9 October 2008 (UTC)

:Either, they're the same thing. You can think of it as being tilted 177 degrees and rotating in the usual direction ("[[prograde]]"), or tilted 3 degrees and rotating backwards ("[[retrograde]]"). It doesn't really make any difference which you go with. (Although, interesting, [[Venus|our article]] says 177 degrees and retrograde, which doesn't sound right to me...) --[[User:Tango|Tango]] ([[User talk:Tango|talk]]) 22:36, 9 October 2008 (UTC)

::Another way to view it is that "retrograde" means the axial tilt is greater than 90°, so "177° and retrograde" is a perfectly legitimate choice and in fact is probably the one I see most often. I don't know how professional astronomers describe it, though. --Anonymous, 01:11 UTC, October 10, 2008.

:Then if Venus is rtro, then count it as 2.7 deg tilt. What about Pluto (which is no longer plnaet). If Pluto is rtro would it still be 120 deg. tilt?--[[User:Freewayguy|<font color="orange">Freeway</font>]][[User talk:Freewayguy|<font color="#FF0000">9</font>]][[Special:Contributions/Freewayguy|<font color="FF0000">1</font>]] 22:51, 9 October 2008 (UTC)

== Quantum Mechanics ==

Since quantum mechanics can't provide deterministic predictions, is it really falsifiable? And if it's not falsifiable, is it really a true science? [[Special:Contributions/12.10.248.51|12.10.248.51]] ([[User talk:12.10.248.51|talk]]) 19:02, 9 October 2008 (UTC)
:The statistical predictions of QM are very much deterministic in the important ways, similar to [[Meteorology]]. In the place QM says things can not be determined ([[uncertainty principle]]) the theory puts a tight limit on the indeterminance(uncertainty) of its predictions. This aspect of the theory would easily be falsified by a theory being able to make a more accurate prediction. It might interest you to know that the prediction of QM explain many aspects of observations we make with chemical instrumentation. If we saw something different than what we do with [[NMR]], [[EPR]], UV-Vis, [[FTIR]], [[Electroanalytical chemistry]] we would have to work on QM. This isn't to say QM is complete its just our best working theory. I've moved this to the reference desk since its a more appropriate place for the question and will receives explanations that far surpass this one.--[[User:OMCV|OMCV]] ([[User talk:OMCV|talk]]) 23:32, 9 October 2008 (UTC)

:You can certainly falsify all sorts of aspects of it, even the stuff that appears almost purely philosophical at first glance. See, for example, the [[Bell test experiments]]. And the non-deterministic nature doesn't preclude testing or even predictions. Simple example: we have no way of making a "deterministic prediction" about when a single unstable atom will decay. But we can make statistical predictions which are easily testable (and thus come up with things like [[half-lives]] which are pretty iron-clad on the aggregate, even if they tell us nothing about the individual atom).
: Note of course that Quantum Mechanics is the name of a field, not the name of a specific theory. Asking "can you test Quantum Mechanics?" is like asking "can you test Biology?" or "can you test Anthropology?" You wouldn't say a field was untestable unless the field's very definition excluded the possibility of naturalistic explanations (like Creationism). You can falsify all sorts of aspects to the theories that make up QM—it's been done since the 1930s, and is why we have the version of QM we have today and not many of the other versions that have been floated. (It's why we have something more like Bohr's version of QM rather than Einstein's, for example.)
:There are some aspects of QM which are at the moment not falsifiable, in the realm of the metaphysical/philosophical interpretations of them. Whether those interpretations count as "science" would depend on who you asked and how you interpreted the term "science" in this case. But even then, sometimes they actually turn out to be falsifiable, like the issue of the [[EPR paradox]], which was thought to be just a philosophical debate with no real testable content when it occurred in the 1930s, but in the 1970s it was discovered that there were very complicated but clever experimental setups that should be able to distinguish between some of the possible answers of it (the Bell tests I linked to earlier). --[[Special:Contributions/98.217.8.46|98.217.8.46]] ([[User talk:98.217.8.46|talk]]) 23:58, 9 October 2008 (UTC)

= October 10 =

== Particle Physics ==

What is the difference between the Higg's boson and the graviton? They both are hypothesized to explain gravity, right? So how are they different?
[[User:Zrs_12|Ζρς ι'β']] <sup><u>[[User_talk:Zrs_12|¡hábleme!]]</u></sup> 00:44, 10 October 2008 (UTC)

:The [[Higgs boson]] is the unobserved member of the [[Standard Model]] that is supposed to explain [[inertial mass]]. The Standard Model does not deal with [[gravity]] at all. The [[graviton]] is the hypothetical force carrier for gravity within various theories of [[quantum gravity]]. [[User:Dragons flight|Dragons flight]] ([[User talk:Dragons flight|talk]]) 01:29, 10 October 2008 (UTC)

::That's right. Sorry, I got the ideas cross for a second. Higgs deals with mass and the standard model, and graviton deals with gravity and hasn't anything to do with the standard model, really. Thanks, [[User:Zrs_12|Ζρς ι'β']] <sup><u>[[User_talk:Zrs_12|¡hábleme!]]</u></sup> 02:19, 10 October 2008 (UTC)

== Turning off all electronic equipment during take-off and landing ==

Why are airline passengers instructed to turn off all electronic equipment during take-off and landing, even equipment that does not contain radio transmitters or receivers? I overheard a conversation recently, in which a fellow passenger claimed that it is done to ensure that people pay attention to what is being said over the loudspeakers, in case of emergencies during the most critical parts of a flight. Can anyone confirm this, or suggest other reasons for this requirement? --[[User:NorwegianBlue|NorwegianBlue]]<sup>[[User_talk:NorwegianBlue|&nbsp;<u>talk</u>]]</sup> 11:07, 10 October 2008 (UTC)

:I've heard the same reason (on numerous ocassions) as you suggest. LIke you say it ensures people are not distracted if there is a need to make an annoucement/emergency decisions. I have been told to stop reading my book before so I would suggest it is more about paying attention than it is about anything else. [[Special:Contributions/194.221.133.226|194.221.133.226]] ([[User talk:194.221.133.226|talk]]) 11:19, 10 October 2008 (UTC)
:In the past it could have been do to with interference (even without transmitters any electronic equipment will emit some EM, I believe), but I'm pretty sure all critical systems on planes are shielded these days. As such, it is probably just to make sure people pay attention and, if not, at least don't make too much noise stopping other people from hearing announcements. On a related note, the reason you aren't allowed to use mobile phones in hospitals is simply because it annoys people, it's been a long time since medical equipment was sensitive to such things. --[[User:Tango|Tango]] ([[User talk:Tango|talk]]) 11:21, 10 October 2008 (UTC)

::In general it's both. EM interference is a legitimate risk (though a much smaller one than when the rules were written in the 60s and 70s), and it is easier to swtich off all electronics than have flight attendents try to figure out which ones actually need to be disabled. At the same time, the [[FAA]] also cites the "possibility of missing important safety announcements during these important phases of flight" [http://ntl.bts.gov/DOCS/91-21_1.html] as an additional reason to turn off electronics during takeoff and landing. [[User:Dragons flight|Dragons flight]] ([[User talk:Dragons flight|talk]]) 11:32, 10 October 2008 (UTC)

::Note as well that handheld electronics represent dangerous projectiles in the cabin in the event of a crash. Headphone cables can present a tripping hazard. On takeoff and landing, the cabin crew want you to stow ''everything'' securely, not just electronics. [[User:TenOfAllTrades|TenOfAllTrades]]([[User_talk:TenOfAllTrades|talk]]) 13:30, 10 October 2008 (UTC)

:The turning off electronics thing is just to "make sure", but realistically there's no point. If turning on an electronic device could really interfere with the cockpit's electronics, then terrorists would have a field day. [[Special:Contributions/98.221.85.188|98.221.85.188]] ([[User talk:98.221.85.188|talk]]) 14:41, 10 October 2008 (UTC)

::The initial justification, [[Crossair Flight 498]], was pretty lame since there were other confounding factors involved. That said, I can hear my speakers making odd noises when I point my cell phone at them the right way, and if I were talking to a control tower to avoid smacking into somebody at 400 knots, I think I'd rather the pilot have a clear signal. [[User:Somedumbyankee|SDY]] ([[User talk:Somedumbyankee|talk]]) 14:51, 10 October 2008 (UTC)
:::Your speakers (and the cables attached to them) aren't shielded from EM interference, I would hope the flight deck radio is. --[[User:Tango|Tango]] ([[User talk:Tango|talk]]) 15:10, 10 October 2008 (UTC)
::::How does that work with wireless communication, though? Then again, I'd imagine that the cell phone bands are all quite separate from the bands that aircraft use. [[User:Somedumbyankee|SDY]] ([[User talk:Somedumbyankee|talk]]) 15:18, 10 October 2008 (UTC)
:::::Is anything in planes wireless? The computers they use for duty free transactions might be, but that's hardly a critical system! --[[User:Tango|Tango]] ([[User talk:Tango|talk]]) 15:27, 10 October 2008 (UTC)
::::::Many planes have satellite radios, satellite TV, etc. for the passengers. Not to mention all of their telemetry equipment that is used to monitor where the plane is, how it is flying, etc. by flight control. --[[Special:Contributions/98.217.8.46|98.217.8.46]] ([[User talk:98.217.8.46|talk]]) 15:49, 10 October 2008 (UTC)

The thinking is that if some of the electronic equipment onboard had been stripped of shielding (say, by shoddy maintenance) then your electronics could interfere. Of course, the plane has a high-voltage radio of its own, which would produce a thousand times more interference than your iPod. It is a dumb rule, but lots of these FAA rules are. They are rituals meant to make you feel safe, not actual safety measures. The lifejackets are a great example. How long do they spend teaching you how to put on a lifejacket? "Your life jacket is located under your seat, or under the arm rest between the seats. Pull the life jacket over your head and attach the strap. Infant life jackets will be distributed, if required. Do not inflate your jacket until you leave the aircraft. Pull the strap until the jacket is properly adjusted. If the life jacket does not inflate or needs more air, blow through the rubber tube." It's a nice image, you bobbing safely in the water with a bright yellow life jacket on. How many people have they actually saved? Zero. Meanwhile hundreds of people die from smoke inhalation which can be prevented by a lightweight mask. There is no rhyme or reason. [[User:Plasticup|<b><font color="#0080FF">Plasticup</font></b>]] [[User_Talk:Plasticup |<font color="#2A8E82"><sup><small>T</small></sup></font>]]/[[Special:Contributions/Plasticup|<font color="#2A8E82"><small>C</small></font>]] 16:05, 10 October 2008 (UTC)

:Are you sure of that number? I'm aware of several water landings where there were survivors; are you saying that in none of the cases were life vests used? --[[User:Carnildo|Carnildo]] ([[User talk:Carnildo|talk]]) 22:32, 10 October 2008 (UTC)
::They shouldn't have been used if the evacuation went as planned since everyone would be in inflatable life rafts. Of course, if you're making a water landing, things aren't exactly going to plan, so... --[[User:Tango|Tango]] ([[User talk:Tango|talk]]) 23:14, 10 October 2008 (UTC)
::Carnildo, for my interest, could you point to a water landing where there were survivors? My impression is that ''no'' commercial (large) jet passengers have ever survived a water impact. Skidding off runways, yes, but not "crashes". I'd be interested in the details. [[User:Franamax|Franamax]] ([[User talk:Franamax|talk]]) 00:58, 11 October 2008 (UTC)

:::See [[Ditching#Survival Rates of Passenger Plane Water Ditchings]]. From the article [http://www.youtube.com/watch?v=3zuLP-QYiy0&feature=related this crash]] had 52 survivors. - [[User:Akamad|Akamad]] ([[User talk:Akamad|talk]]) 02:19, 11 October 2008 (UTC)

::::And more specifically, [[Ethiopian Airlines Flight 961]], although I'm under the impression that life jackets actually killed more people than they saved in that particular incident. --[[User: Antilived|antilived]]<sup>[[User_talk:Antilived|T]] | [[Special:Contributions/Antilived|C]] | [[User:Antilived/Gallery|G]]</sup> 05:12, 11 October 2008 (UTC)

:coincidentally, yesterday:
:''Safety investigators will now ask passengers if they were using any electronic equipment at the time of this latest incident. "Certainly in our discussions with passengers that is exactly the sort of question we will be asking - 'Were you using a computer?'," The Courier Mail quoted an Australian Transport Safety Bureau (ATSB) spokesman as saying. The ATSB said the pilots received messages about "some irregularity with the aircraft's elevator control system", before the plane climbed 300 feet and then nosedived.'' [http://www.nzherald.co.nz/world/news/article.cfm?c_id=2&objectid=10536660] but apparently they've decided laptops were innocent.
:that article does contain the following surprising (to me) sentence, though: ''In July, a passenger clicking on a wireless mouse mid-flight was blamed for causing a Qantas jet to be thrown off course, according to the Australian Transport Safety Bureau's monthly report.'' [[User:Gzuckier|Gzuckier]] ([[User talk:Gzuckier|talk]]) 05:33, 11 October 2008 (UTC)

::Thanks, everyone, for your responses! --[[User:NorwegianBlue|NorwegianBlue]]<sup>[[User_talk:NorwegianBlue|&nbsp;<u>talk</u>]]</sup> 12:52, 11 October 2008 (UTC)

== problem sum ==

a constant retarding force of 50 newtons is applied to a body of mass 20 kilograms moving initially with speed of 15 metres per second. how lomg does the body take to stop <span style="font-size: smallest;" class="autosigned">—Preceding [[Wikipedia:Signatures|unsigned]] comment added by [[Special:Contributions/77.31.137.209|77.31.137.209]] ([[User talk:77.31.137.209|talk]]) 16:22, 10 October 2008 (UTC)</span><!-- Template:UnsignedIP --> <!--Autosigned by SineBot-->

:Not that we should solve homework problems for you, but consider that a newton is a kilogram meter per second squared (kg*m/s<sup>2</sup>) and simple [[Units conversion by factor-label|factor label cancelling]] (i.e. do the algebra with the units to figure out how to multiply and divide the numbers) should give you the answer. The article I linked shows the basic framework for solving problems like this. --[[User:Jayron32|Jayron32]].[[User talk:Jayron32|<small>talk</small>]].[[Special:Contributions/Jayron32|<small>contribs</small>]] 16:51, 10 October 2008 (UTC)
::While I would probably do the same as you, the more standard approach is the learn the constant acceleration formulae. Pick the appropriate one of them along with F=ma and substitute in the numbers. --[[User:Tango|Tango]] ([[User talk:Tango|talk]]) 16:58, 10 October 2008 (UTC)
:::True, but that requires one to either memorize a list of formlas, or to be able to work the calculus on one formula to derive the rest. The nice thing about the factor-label method is that it requires learning a single method that is broadly applicable accross MANY fields. It will get you the right answer, for example, in any high school or introductory collegiate chemistry and/or physics class for, quite literally, 90% of the homework problems you will get. One method, 90% of the problems. The "learn every formula method" also works, but is, IMHO, more labor intensive and time consuming. --[[User:Jayron32|Jayron32]].[[User talk:Jayron32|<small>talk</small>]].[[Special:Contributions/Jayron32|<small>contribs</small>]] 17:33, 10 October 2008 (UTC)
::::Couldn't agree more, but it's generally best to help someone get to grips with the method they're being taught (which is almost certainly the memorise formulae method) rather than teaching them a whole new method. --[[User:Tango|Tango]] ([[User talk:Tango|talk]]) 19:01, 10 October 2008 (UTC)

== threshold logic synthesis ==

can anybody help me to know how to find the false vertices for threshold logic synthesis....... <small><span class="autosigned">—Preceding [[Wikipedia:Signatures|unsigned]] comment added by [[User:Sveta rathi|Sveta rathi]] ([[User talk:Sveta rathi|talk]] • [[Special:Contributions/Sveta rathi|contribs]]) 19:04, 10 October 2008 (UTC)</span></small><!-- Template:Unsigned --> <!--Autosigned by SineBot-->

:Wikipedia has an article on [[Logic synthesis]] that also has a long list of referenes and other links at the end. This may be a good place to start. --[[User:Jayron32|Jayron32]].[[User talk:Jayron32|<small>talk</small>]].[[Special:Contributions/Jayron32|<small>contribs</small>]] 19:20, 10 October 2008 (UTC)

== "Virgin birth" in a shark ==

http://ap.google.com/article/ALeqM5gV-UePymWuPU7HFxNgUXRUrakU1wD93NPTM80

How did this exactly happen? Can the same thing happen to humans? --[[User:Emyn ned|Emyn ned]] ([[User talk:Emyn ned|talk]]) 20:07, 10 October 2008 (UTC)

:The process is called [[parthenogenesis]]. (Our article discusses the matter in some detail.) Briefly, there's never been a substantiated case of parthenogenesis in any mammal (including humans) in the wild. Induced parthenogenesis has apparently been demonstrated in rabbits and mice, producing viable offspring. Human parthenogenesis has been demonstrated to the extent of creating human embryonic stem cells from unfertilized eggs, though no human beings have been born via this method. [[User:TenOfAllTrades|TenOfAllTrades]]([[User_talk:TenOfAllTrades|talk]]) 20:22, 10 October 2008 (UTC)

:it's not too hard to make an egg cell of any animal start to divide, even if not fertilized; it's all caused by a bunch of calcium entering, calcium being a handy ion in organisms for signals like that, it's not too scarce like magnesium and not too abundant like sodium. The chromosomes provided by the sperm actually don't have anything to do with triggering the egg's development, but the whole thing is set up that the arrival of the sperm triggers that calcium influx which starts the division, and the arrival of the chromosomes in the same package is basically a happy accident. but you can trigger the calcium influx via drugs and things in eggs of many species and away they go. once the cell starts the process of dividing and then duplicating chromosomes and dividing again, the fact that it only has one set of chromosomes instead of two gets fixed as that set gets duplicated after a cell division. (the other product of the division has no sets of chromosomes, and just sort of fades away, but that's not a problem; cells early in the division process aren't specialized yet, each one can produce a complete embryo if they're separated, that's where identical twins come from.) so at that point you've got a dividing egg cell with two sets of chromosomes, and you're on your way. the fact that both sets are identical isn't critical, but of course any undesirable recessive genes will pop up.

:in sharks? well we're seeing a lot of it now, because we didn't look for it before, so it's likely that some mechanism has evolved which can trigger the egg reasonably frequently without the need for sperm. there are other species which don't have males at all, of course, so this is just a less emphatic version of that. might happen occasionally in other animals too; people for instance. it would be hard to prove, particularly if it was pretty rare. there is this one story about a virgin, a couple of thousand years ago.... [[User:Gzuckier|Gzuckier]] ([[User talk:Gzuckier|talk]]) 20:29, 10 October 2008 (UTC)

:It could happen in humans, I believe. Unfortunately or fortunately, only females would be produced in such cases, due to the XY sex chromosome thing. [[User:Imagine Reason|Imagine Reason]] ([[User talk:Imagine Reason|talk]]) 04:15, 11 October 2008 (UTC)

== i really should know this but ==

how do volumes add in a liquid solution? i know all about partial pressures, etc. but if 10 ml of alcohol is added to 90 ml of water, is the result 100 ml? how about when dissolving solids in a liquid? thanks. [[User:Gzuckier|Gzuckier]] ([[User talk:Gzuckier|talk]]) 20:18, 10 October 2008 (UTC)

:Unfortunately, there's not a simple answer to this one. In general, the volume of a solution will ''not'' be equal to the sum of the separate volumes of its components. In other words, if you add 10 mL of ethanol to 90 mL of water, the final volume will come out to be slightly less than 100 mL (about 99.5 mL, actually). This discrepancy will depend on the compounds being mixed, and on their proportions. (If you add 40 mL of ethanol to 60 mL of water, the final solution will be a shade less than 98 mL volume.) Dissolving solids in liquids has similar problems.

:Conceptually, you can think of the molecules of solute being able to at least partially occupy gaps left between the loosely-packed solvent molecules, but that's an awfully hand-waving description. [[User:TenOfAllTrades|TenOfAllTrades]]([[User_talk:TenOfAllTrades|talk]]) 20:44, 10 October 2008 (UTC)

::As a more detailed description, consider that there is in the individual pure liquids, the macroscopic "volume" property is determined by a microscopic property we can call "intermolecular distance" that the molecules seperate themselves by. So, there is a water-water intermolecular distance and a ethanol-ethanol intermolecular distance. When you mix the two, you create a new interaction, the ethanol-water interaction, which is a shorter distance than either the ethanol-ethanol or water-water distance. This makes sense if you consider that in order to for two substances be miscable, the molecules of each substance must be more attracted to ''each other'' than to themselves.(if the water and ethanol were more attracted to themselves than to each other, then the two would merely aggregate seperately, and would not mix). More attractive force means shorter distance between them. So a solution of two substances should always occupy a smaller volume than the sum of their pre-mixed volumes. --[[User:Jayron32|Jayron32]].[[User talk:Jayron32|<small>talk</small>]].[[Special:Contributions/Jayron32|<small>contribs</small>]] 23:13, 10 October 2008 (UTC)

:::That's a completely different description. The first one is wrong, and just happens to partially explain a little. — [[User:DanielLC|DanielLC]] 16:22, 11 October 2008 (UTC)

== What is the movement of a submarine called? ==

Sailing? Driving? <span style="font-size: smaller;" class="autosigned">—Preceding [[Wikipedia:Signatures|unsigned]] comment added by [[Special:Contributions/87.165.220.170|87.165.220.170]] ([[User talk:87.165.220.170|talk]]) 20:55, 10 October 2008 (UTC)</span><!-- Template:UnsignedIP --> <!--Autosigned by SineBot-->

:'Sailing' appears to be a widely-used, widely-accepted term. (Google ''submarine sailed'' or ''submarine sailing'' to see many, many examples of usage.) 'Driving' is definitely not. [[User:TenOfAllTrades|TenOfAllTrades]]([[User_talk:TenOfAllTrades|talk]]) 23:30, 10 October 2008 (UTC)

::It's odd that, for ships, at one time "sailing" was updated to "steaming" but now seems to have reverted to "sailing" even though the technology has gone somewhere else entirely. I guess dieseling, electricking or nuclear reacting just don't roll off the tongue. [[User:Spinningspark|<font style="background:#FFF090;color:#00C000">'''Sp<font style="background:#FFF0A0;color:#80C000">in<font style="color:#C08000">ni</font></font><font style="color:#C00000">ng</font></font><font style="color:#2820F0">Spark'''</font>]] 12:41, 11 October 2008 (UTC)

::''So we sailed up to the sun . . . in our yellow submarine'' <small>- Lennon-McCartney</small>

== Sudden moon ==

I am writing a story set on a planet that has suddenly acquired a moon. For the purposes of this question, just assume it suddenly appeared and that both the planet and moon are the same size as ours.

I'm trying to work out exactly the nature and scale of the disasters this would cause. All I've got so far are extreme tidal waves and flooding, but would there also be earthquakes? I think it would probably result in massive unbalancing and possibly death among nocturnal species, not to mention the probable extinction of a lot of tidal zone species. What else, though?

While I'm at it, what would a world be like that did not have a moon? It would have very small tides thanks to the sun, and would always be very dark at night... One of my friends claims that it wouldn't have seasons, but I find that dubious. Any thoughts? --[[User:Masamage|Masamage]] [[User talk:Masamage|♫]] 21:12, 10 October 2008 (UTC)

:I don't think you would get anything more drastic that we get every day, since as the Earth rotates different parts of the Earth are affected by the moon's gravity (you might get some problems immeadiately after the arrival since you would have the effect of 6 hours worth of tidal change in an instant [although the magnitude of the tides would be the same as for the Earth], but that wouldn't last long, although the damage from it might). The fact that life wouldn't be used to the tides would be a problem, certainly, but I think that's about it (and remember, it's just increased tides, not new tides, because of the sun). As for a planet without a moon, it would probably affect the seasons, but it wouldn't preclude having them. Seasons are caused by the rotational axis being tilted with respect to the orbit, the moon may well have affected our axial tilt, but having a moon isn't a requirement to have one. The day would also be shorter, since the same tidal forces which mean the moon always shows the same face to Earth are gradually slowing the Earth's rotation, without the moon that wouldn't have happened so the day would be a few hours shorter (I'm not sure how many, but I believe it's been measured by looking at fossilised coral). I read somewhere that the moon has helped stabilise the Earth's rotational axis, but I'm not sure how, so the seasons may be more variable without a moon (although probably on the scale of centuries at least). --[[User:Tango|Tango]] ([[User talk:Tango|talk]]) 21:31, 10 October 2008 (UTC)

:Instantaneous appearance? You'd get at least the following:
:# A sudden shock as the planet shifts from following a simple orbital path around its sun to the sine-wave pattern the Earth follows. (The center of gravity of the Earth-Moon system follows an elliptical path around the Sun; the Earth and Moon orbit that center of gravity with a period of one Lunar month).
:# Gradually-increasing tidal heights, with the final tidal range being about three times what it used to be. Timing of the high tides will also change. You won't get tidal waves because the water has a long way to flow to adapt to the new gravity patterns, and it doesn't move very fast. At a guess, it'll take a month or two for the tides to reach their final heights.
:# More earthquakes. They won't be stronger, and they might be weaker, because of increased tidal flexing of the crustal plates.
:# Disruption of activity for many species: some night-active species will have trouble being active during full moons; some day-active species will stay active at night during full moons.
:# Increased predation of day-active prey species. The increased night-time lighting means that camoflage patterns and sleeping habits are no longer adequate for protection. It'll take about five years for the resulting boom-and-bust of night-active predator species to settle out.
:Over the long term, you'd see the following:
:# More reliable seasons. Adding a large moon will stabilize the planet's rotation axis, so the strength of the seasons won't change over time (millions to billion of years)
:# An increase in the number of night-active species. More light means it's easier to move around at night. (hundreds to tens of thousands of years)
:# A reduction in meteor impacts: the Moon provides some protection against meteors by variously blocking them, causing them to break up, or throwing them out of Earth-crossing orbits.
:# An increase in volcanism: the increase in tidal flexing will increase the temperature of the planet. (Hundreds of thousands to millions of years)
:Hope this helps. --[[User:Carnildo|Carnildo]] ([[User talk:Carnildo|talk]]) 22:58, 10 October 2008 (UTC)

::If you are interested in this subject, and want to see how another author has treated a similar situation (though not exactly the same), may I recommend [[Jack McDevitt]]'s book '''Deepsix''', it has some interesting descriptions of weird tidal effects caused by two large planets on a collision course. Its part of his "Priscilla Hutchins" series, and is an enjoyable read. --[[User:Jayron32|Jayron32]].[[User talk:Jayron32|<small>talk</small>]].[[Special:Contributions/Jayron32|<small>contribs</small>]] 23:05, 10 October 2008 (UTC)

::Why would it take months for the tides to reach their maximum? The water doesn't need to move any faster than it does on Earth and it can go all the way round the Earth in 24 hours (well, individual bits of water don't, but you know what I mean). --[[User:Tango|Tango]] ([[User talk:Tango|talk]]) 23:13, 10 October 2008 (UTC)

:::On Earth, the tides have had billions of years to build up momentum. The Moon's gravity isn't very strong, so the water doesn't accelerate very fast. --[[User:Carnildo|Carnildo]] ([[User talk:Carnildo|talk]]) 23:29, 10 October 2008 (UTC)

::::That's utter nonsense.

::::The water oscillates back and forth twice per day. So the momentum builds up slowly over about six hours then back the other way over the next six. It doesn't "build up" over millions of years. The tides would settle into their regular pattern in about a day or two. I think there could be no earthquakes or anything because the earth too squeezes and stretches in a cycle over 12 hours and we don't see particular problems because of that. I agree though that the SUDDEN arrive of a few gigatons of stuff in orbit would very abruptly jerk the planet in it's orbit - and that would be utterly disasterous - all of the oceans and atmosphere would slosh violently - possibly flying off the planet completely...it's hard to imagine any life surviving that. But if the moon somehow slowly spiralled into position over decades - then I think it would have fairly benign effects (well, crazy weather - tides where they'd never been before - flooding, rivers running backwards...but definitely something you could survive). You'd also have to consider the consequences of the moon suddenly being dumped into the much stronger gravity well of the planet. Since we have no knowledge of the "magic" that makes the moon suddenly teleport into place - we can't guess what forces that entails - but it might well break up, ending up as a truly spectacular ring system...or possibly raining death and destruction onto the surface. [[User:SteveBaker|SteveBaker]] ([[User talk:SteveBaker|talk]]) 01:12, 11 October 2008 (UTC)

== hydrogen reaction ==

Will a balloon filled with pure hydrogen and pierced with a needle made of palladium explode/combust? <small><span class="autosigned">—Preceding [[Wikipedia:Signatures|unsigned]] comment added by [[User:Kaufmann1|Kaufmann1]] ([[User talk:Kaufmann1|talk]] • [[Special:Contributions/Kaufmann1|contribs]]) 21:57, 10 October 2008 (UTC)</span></small><!-- Template:Unsigned --> <!--Autosigned by SineBot-->
:Not unless the palladium is particularly hot. Palladium can act as a catalyst for certain reactions involving hydrogen, however as far as I am aware, it does not lower the [[activation energy]] of the combustion reaction enough to cause it to become spontaneous. --[[User:Jayron32|Jayron32]].[[User talk:Jayron32|<small>talk</small>]].[[Special:Contributions/Jayron32|<small>contribs</small>]] 22:56, 10 October 2008 (UTC)

:Pure hydrogen? What would it react with? --[[User:Carnildo|Carnildo]] ([[User talk:Carnildo|talk]]) 22:58, 10 October 2008 (UTC)
::The oxygen in the atmosphere outside the balloon, presumably. [[User talk:Algebraist|Algebraist]] 23:05, 10 October 2008 (UTC)

:Our article on palladium says it can absorb large amounts of hydrogen. I'm not sure if that reaction releases energy, but I doubt it would do so to the extent of causing an explosion. --[[User:Tango|Tango]] ([[User talk:Tango|talk]]) 23:10, 10 October 2008 (UTC)

== Mars climate ==

How does climates on Mars work?Does temperats range by latitudes, seasons or night-and day. Ithouhgt Mars is a very cold planet, often colder than a freezer, and the average planet temp is around minus 67 F. Is that the mid-latitude average surface temp? I thought only tropical zones of Mars or low latitudes get temp range from +10 to +69 F.--'''[[User:Freewayguy|<font color="orange">Freeway</font>]][[User talk:Freewayguy|<font color="#FF0000">9</font>]][[Special:Contributions/Freewayguy|<font color="FF0000">1</font>]]''' 22:15, 10 October 2008 (UTC)
:Interestingly, we have an article on that: [[Climate of Mars]]. --[[User:Jayron32|Jayron32]].[[User talk:Jayron32|<small>talk</small>]].[[Special:Contributions/Jayron32|<small>contribs</small>]] 22:58, 10 October 2008 (UTC)

: The variation of temperature with latitude, seasons and day/night don't really depend on the planet so much as how it moves around the sun. Pretty much all of the planets have all of those things <s>(with the sole exception of Mercury which keeps the same face pointing towards the sun all the time - so it doesn't have day/night cycles)</s>.
:* Variation by latitude is because the planet is round and the sun's rays spread out more at the poles than at the equator.
:* Variation by season is because most planets are doing their daily rotation about an axis that's tipped over somewhat. This means that the suns rays are more spread out at some times of the year than others. For planets with very elliptical orbits, there is a variation due to distance from the sun too.
:* Variation between night and day is because the sun isn't shining on the surface at night.
: So all of those things vary on all planets that are round, have an axial tilt and rotate on their axis...and that includes Mars.
: [[User:SteveBaker|SteveBaker]] ([[User talk:SteveBaker|talk]]) 00:47, 11 October 2008 (UTC)
::Actually, Mercury isn't tidally locked (although astronomers did think it was at one time). See [[Mercury (planet)#Spin–orbit resonance]]. --[[User:Tango|Tango]] ([[User talk:Tango|talk]]) 00:55, 11 October 2008 (UTC)
::Seconded. Apparently Steve is stuck in 1964;-) --[[User:Stephan Schulz|Stephan Schulz]] ([[User talk:Stephan Schulz|talk]]) 01:01, 11 October 2008 (UTC)
:::In more ways than you can imagine! [[User:SteveBaker|SteveBaker]] ([[User talk:SteveBaker|talk]]) 01:20, 11 October 2008 (UTC)

So isn't temperate zone on Mars lattide of 30+ always or often below 0, and lattitude of 50+ alwas colder than Greenland? Generally, Mars I thought is very cold.--'''[[User:Freewayguy|<font color="orange">Freeway</font>]][[User talk:Freewayguy|<font color="#FF0000">9</font>]][[Special:Contributions/Freewayguy|<font color="FF0000">1</font>]]''' 01:04, 11 October 2008 (UTC)
:Because of its thin atmosphere it doesn't retain heat well, so at night it is going to be extremely cold regardless of your latitude. During the day, it will be warmer the nearer the equator you are (well, not quite the equator due to the axial tilt). I don't know any numbers off the top of my head, but they shouldn't be too difficult to find with a bit of googling. --[[User:Tango|Tango]] ([[User talk:Tango|talk]]) 13:53, 11 October 2008 (UTC)

== gas constant? ==

why should we use a constant for gas equations?where does the universal gas constant come from?i couldn't find any information about history of gas constant(R)? <span style="font-size: smaller;" class="autosigned">—Preceding [[Wikipedia:Signatures|unsigned]] comment added by [[Special:Contributions/88.242.106.180|88.242.106.180]] ([[User talk:88.242.106.180|talk]]) 00:57, 11 October 2008 (UTC)</span><!-- Template:UnsignedIP --> <!--Autosigned by SineBot-->

:You may want to have a look at [[gas constant]] and [[Boltzmann constant]] for a more detailed treatment of the topic. Briefly, the gas constant (R) is a proportionality constant which describes how much energy is stored in a mole of (ideal) gas molecules per degree of temperature. (The related Boltzmann constant, k<sub>B</sub>, describes the quantity of energy per molecule.) [[User:TenOfAllTrades|TenOfAllTrades]]([[User_talk:TenOfAllTrades|talk]]) 01:40, 11 October 2008 (UTC)

::A pity nothing is said in the articles about experiments like in [[:de:Universelle_Gaskonstante#Ein Experiment zur Ermittlung einer Näherung der Gaskonstante]], or how the constant was measured to this accuracy. --[[User:Ayacop|Ayacop]] ([[User talk:Ayacop|talk]]) 09:26, 11 October 2008 (UTC)

:::If you can translate from the german, [[WP:SOFIXIT|the English Wikipedia articles could probably benefit from your help]]. --[[User:Jayron32|Jayron32]].[[User talk:Jayron32|<small>talk</small>]].[[Special:Contributions/Jayron32|<small>contribs</small>]] 12:46, 11 October 2008 (UTC)

:Back to the gas constant. The SI system was carefully constructed to in general, avoid these sort of proportionality constants. Many calculations would require them, except that the units are defined to be compatable in ways that generate proprotionality constants of "1". The situation with "R" is because the SI unit for temperature, [[kelvin]], is created not to be compatable with other SI units, but be compatable with the [[Celsius]] scale. Since the size of a Celsius unit is arbitary (there's nothing inherently useful about being 1/100th the difference between the sea-level freezing and boiling points of water), the size of the kelvin is arbitrary as well. One could define a temperature scale where 1 degree was equal to the the amount of energy contained by 1 mole of molecules, and under THAT scale, R would be equal to 1. However, for other reasons of convenience and history, we use the Kelvin scale, so we are stuck with a non-unitary R values. --[[User:Jayron32|Jayron32]].[[User talk:Jayron32|<small>talk</small>]].[[Special:Contributions/Jayron32|<small>contribs</small>]] 13:00, 11 October 2008 (UTC)

::I'm not entirely sure I'd agree with that. While interconversion among SI units is very straightforward and generally avoids weird proportionality constants, such constants are almost always necessary in calculations which describe physical processes in the real world. (The energy of a photon is equal to its frequency multiplied by 6.626x10<sup>-34</sup>: the [[Planck constant]]; the gravitational attraction between two bodies is the product of their masses divided by the square of their separation distance, multipied by 6.674x10<sup>-11</sup>: the [[gravitational constant]]. And so forth.)
::The seven [[SI base unit|base SI units]] trace their roots to essentially arbitrary roots which don't have any universal scientific or physical significance. (The meter was originally based on a rough measure of the Earth's circumference; the second on arbitrary divisions in the length of Earth's day; the kilgram tied to the density of water.)
::To get rid of arbitrary constants of proportionality, physicists will resort to systems of so-called ''[[natural units]]'' which peg most physical constants to be exactly 1. Under (for example) [[Planck units]], the speed of light, the gravitational constant, the reduced Planck's constant, Boltzmann's constant, and the Coulomb force constant are all set to be 1, and other units defined from there. Such systems can make calculations dramatically 'neater' and eliminate the risk of 'losing' a constant in a complicated expression. The downside of such systems is that they generate base units which aren't convenient for 'everyday' usage. (The base unit of temperature in Planck units is about 10<sup>32</sup> kelvin, and the base unit of time is about 10<sup>-44</sup> seconds.) [[User:TenOfAllTrades|TenOfAllTrades]]([[User_talk:TenOfAllTrades|talk]]) 14:51, 11 October 2008 (UTC)

== Use of Oil ==

Generally what percentage of a barrel of oil is used strictly for fuels such as gasoline and diesel? How much is used for plastics and other products? I had heard that oil used for fuels was low - around 20% - and the bulk of every oil pumped out of the ground was for other products like plastic. Is this true[[Special:Contributions/142.68.216.154|142.68.216.154]] ([[User talk:142.68.216.154|talk]]) 02:35, 11 October 2008 (UTC)

:You might want to look at this link [https://eed.llnl.gov/flow/images/USEnFlow02-quads.gif]. Which deals with oils use for energy. Only 20-30% of the energy we use goes to transportation but almost all of that energy comes from oil. I know that doesn't answer you question but it is probably the origin of you mangled statistic. What comes out of a barrel of oil depends on what the oil is like (where it was found) and how you crack it but this link gives you and idea of how an average barrel gets fractioned [http://www.hislandoil.com/images/barrel.gif]. The key chunks of plastics are mostly derived from natural gas. The other components are derived from side products in process of refining oil for gasoline/diesel. Transportation fuel is the largest and most powerful market for oil, plastic just removes 4.7% of the barrel of what would other wise be a waste stream to burn for heat/electcity or maybe converted into hydrogen. In addition consumers can afford to pay more for natural gas to heat their homes and produce electricity than chemical producers can afford to pay for natural gas as a feed stock. The price of natural gas in North America has forced many chemical producers to close up shop and move to places with cheaper natural gas like the Middle East and Africa. I think BASF cited this when they closed plants around 2005 among other companies. I hope that helps.--[[User:OMCV|OMCV]] ([[User talk:OMCV|talk]]) 03:28, 11 October 2008 (UTC)

:yeah, the demand for gasoline/fuel oil basically requires economically to "crack" as much of the petroleum that can possibly be used into the proper weights. In addition, the advent of fuel injection and the associated in-tank fuel pumps have made it possible to add the lighter petroleum fractions into gasoline which would have created a lot of vapor lock in the carbureted engines with the fuel pump on the engine, and used to be disposed of. In fact, (according to what i read) the vapor pressure on gasoline has risen enough even just in a decade or two to saturate the vapor capture systems on cars from the 80s. basically, any oil that goes into plastics is leftovers that would otherwise be waste. [[User:Gzuckier|Gzuckier]] ([[User talk:Gzuckier|talk]]) 05:22, 11 October 2008 (UTC)

== Plants with edible stems ==

Are there plants other than Rhubarb with edible stems?[[Special:Contributions/74.50.200.72|74.50.200.72]] ([[User talk:74.50.200.72|talk]]) 06:41, 11 October 2008 (UTC)
:[[Cattails]] [[User:Zrs_12|Ζρς ι'β']] <sup><u>[[User_talk:Zrs_12|¡hábleme!]]</u></sup> 07:18, 11 October 2008 (UTC)
:: [[Leek]]s and [[spring onion]]s are commonly eaten in the UK. [[User:Axl|<font color="#808000">'''Axl'''</font>]] <font color="#3CB371">¤</font> <small>[[User talk:Axl|<font color="#6B8E23">[Talk]</font>]]</small> 07:20, 11 October 2008 (UTC)

:::The pedia does it again -- try [[Edible plant stems]] for a nice list of munchies. (It doesn't mention mushroom stems/stalks which are not notable but edible.) [[User:Julia Rossi|Julia Rossi]] ([[User talk:Julia Rossi|talk]]) 07:25, 11 October 2008 (UTC)
::::[[Mushroom]]s are also not [[plants]]. —[[User:Ilmari Karonen|Ilmari Karonen]] <small>([[User talk:Ilmari Karonen|talk]])</small> 07:33, 11 October 2008 (UTC)

:(ec) A lot of [[herb]]s are edible in the whole (or at least their above-ground parts are), so I guess they count. At the other end of the scale, [[pine]] [[phloem]] is edible (if not very nutritious), though the ''whole'' trunk isn't. —[[User:Ilmari Karonen|Ilmari Karonen]] <small>([[User talk:Ilmari Karonen|talk]])</small> 07:31, 11 October 2008 (UTC)

::Rhubarb's culinary cousin [[Celery]] certainly qualifies, doesn't it?--[[User:Jayron32|Jayron32]].[[User talk:Jayron32|<small>talk</small>]].[[Special:Contributions/Jayron32|<small>contribs</small>]] 12:44, 11 October 2008 (UTC)

== Noisy faucet ==

My kitchen faucet, which works well otherwise, makes a high pitched whine when running hot water through it. Why? <span style="font-family:monospace;">[[User:Dismas|Dismas]]</span>|[[User talk:Dismas|<sup>(talk)</sup>]] 15:19, 11 October 2008 (UTC)

== Quote identification ==

With regard to the [[Fermi paradox]]: "If there are so many alien civilizations, why haven't they visited us? I decided to do an experiment. I wanted lobsters for dinner. I put a plate on my table, sat down, opened the front door, and waited for a lobster to crawl onto my plate. Three hours later, no lobster came. I ended the experiment, concluding there are no lobsters in the world."

This "quote", which is obviously not word-for-word, is from a show that aired on Discovery Channel a few years ago. --[[Special:Contributions/99.237.96.81|99.237.96.81]] ([[User talk:99.237.96.81|talk]]) 16:34, 11 October 2008 (UTC)

== would you freeze in space with nothing on ==

if i went into outer space without a suit would i freeze instantly? why? is there no oxygen or something?--[[User:Majorcolors1|Majorcolors1]] ([[User talk:Majorcolors1|talk]]) 17:59, 11 October 2008 (UTC)

== Robotics - what is static stability? ==

I find definition of "static stability" as it relates to missiles and whatnot, but not as it relates to robotics. It sounds like it means just the concept that when a robot is at rest it should be stable, but I don't know for sure so I decided to query the WP community. [[User:Dragon_Smaug|<b><font color="#006500">S</font><font color="#007500">m</font><font color="#008500">a</font><font color="#009500">u</font><font color="#00A500">g</font></b>]] 18:01, 11 October 2008 (UTC)
: Ack nevermind. Just found the answer: "A statically stable robot can stand still without falling over." So I was right. [[User:Dragon_Smaug|<b><font color="#006500">S</font><font color="#007500">m</font><font color="#008500">a</font><font color="#009500">u</font><font color="#00A500">g</font></b>]] 18:02, 11 October 2008 (UTC)

Revision as of 18:03, 11 October 2008

w w w . A n o n T a l k . c o m